You are on page 1of 66

Total Marks : 200

Online Prelims TEST - 13 (SUBJECT WISE)


( InsightsIAS Mock Test Series for UPSC Preliminary Exam 2020 )

1 With reference to the Solar Energy Corporation of India(SECI), consider the following statements:
1. It was originally registered under Section 25 of Companies Act, 1956, as a Company not for profit.
2. It is under the administrative control of the Ministry of Power.
3. It is the only CPSU dedicated to the solar energy sector.

Which of the statements given above is/are correct?


A. 1 only
B. 1 and 3 only
C. 2 and 3 only
D. 1, 2 and 3

Correct Answer : B

Answer Justification :

"Solar Energy Corporation of India ltd" (SECI) is a CPSU under the administrative control
of the Ministry of New and Renewable Energy (MNRE), set up on 20th Sept, 2011 to facilitate
the implementation of JNNSM and achievement of targets set therein. Hence, statement 2 is
incorrect.

It is the only CPSU dedicated to the solar energy sector. Hence, statement 3 is correct.

It was originally incorporated as a section-25 (not for profit) company under the
Companies Act, 1956.

However, through a Government of India decision, the company has recently been converted into a
Section-3 company under the Companies Act, 2013. The mandate of the company has also been
broadened to cover the entire renewable energy domain. Hence, statement 1 is correct.

https://economictimes.indiatimes.com/small-biz/productline/power-generation/solar-pv-cells-manufa
ctured-using-blue-wafers-no-longer-to-be-classified-as-domestically-manufactured-cells-
mnre/articleshow/71724217.cms

2 Among which of the following countries the famous Wakhan National Park is located?

A. Iran
B. Afghanistan
C. India
D. Cambodia

Correct Answer : B

Answer Justification :

Wakhan National Park is a national park in Afghanistan. Established in 2014, the park
comprises the entire district of Wakhan, extending along the Wakhan Corridor between the Pamir

1
Total Marks : 200
Online Prelims TEST - 13 (SUBJECT WISE)
( InsightsIAS Mock Test Series for UPSC Preliminary Exam 2020 )

mountains and the Hindu Kush, bordering Tajikistan to the north, Pakistan to the south, and China
to the east.

Hence, option (b) is correct.

https://www.iucn.org/content/wcs-applauds-afghanistans-declaration-establishing-entire-wakhan-dis
trict-countrys-second

http://seci.co.in/other_page.php?mmid=1&page=about_us

3 With reference to the PACEsetter Fund, consider the following statements:


1. It is a fund set up jointly by India and China.
2. The Fund's main purpose is to improve the viability of off-grid renewable energy businesses that sell
small scale (under 1 megawatt) clean energy systems to individuals and communities without access
to grid connected power or with limited/intermittent access.

Which of the statements given above is/are correct?


A. 1 only
B. 2 only
C. Both 1 and 2
D. Neither 1 nor 2

Correct Answer : B

Answer Justification :

In 2013, the Governments of the Republic of India and the United States of America
launched Promoting Energy Access through Clean Energy (PEACE) as a new track under the
U.S.-India Partnership to Advance Clean Energy (PACE), alongside the existing research (PACE-R)
and deployment (PACE-D) tracks. PEACE aims to harness commercial enterprise to bring clean
energy to individuals and communities un-served and underserved by the electricity grid. In June
2015, these governments established the PACEsetter Fund, a new joint fund to support the
PEACE initiative by providing early-stage grant funding to accelerate the
commercialization of innovative off-grid clean energy products, systems, and business
models. Hence, statement 1 is incorrect.

The Fund's main purpose is to improve the viability of off-grid renewable energy businesses that
sell small scale (under 1 megawatt) clean energy systems to individuals and communities
without access to grid connected power or with limited/intermittent access (less than 8
hours per day). Hence, statement 2 is correct.

http://pacesetterfund.org/home.html

https://www.pib.nic.in/PressReleasePage.aspx?PRID=1585745

4 With reference to the Pobitora Wildlife Sanctuary, consider the following statements:

2
Total Marks : 200
Online Prelims TEST - 13 (SUBJECT WISE)
( InsightsIAS Mock Test Series for UPSC Preliminary Exam 2020 )

1. It is located in the state of Assam


2. It is situated on the bank of river Teesta.
3. It provides habitat for the Indian rhinoceros.

Which of the statement given above is/are correct?


A. 1 and 3 only.
B. 1 only
C. 2 and 3 only
D. 1, 2 and 3

Correct Answer : A

Answer Justification :

Pobitora Wildlife Sanctuary is a wildlife sanctuary on the southern bank of the Brahmaputra in
Morigaon district in Assam, India. Hence, statement 1 is correct and statement 2 is
incorrect.

It was declared in 1987 and covers 38.85 km², providing grassland and wetland habitat for the
Indian rhinoceros. Hence, statement 3 is correct.

https://www.thehindu.com/sci-tech/energy-and-environment/anthrax-scare-in-reserve-after-death-of-
buffaloes/article29752405.ece

5 Which of the following nations are considered as snow leopard countries under the Global Snow
Leopard & Ecosystem Protection Program (GSLEP):
1. India
2. New Zealand
3. Afghanistan
4. Kazakhstan

Select the correct answer using the code given below?


A. 1 and 3 only
B. 1, 3 and 4 only
C. 2 and 4 only
D. 1, 2, 3 and 4

Correct Answer : B

Answer Justification :

The Global Snow Leopard and Ecosystem Protection Program (GSLEP) launched PWAS
(Population Assessment of the World’s Snow Leopards) in 2018 to bring together snow
leopard range countries in an effort to accurately estimate their population size and monitor them.
At present, the number of snow leopards around the world is not known and the existing estimates
may not be accurate. PAWS’s goal is to have a “robust estimate” of the snow leopard population by

3
Total Marks : 200
Online Prelims TEST - 13 (SUBJECT WISE)
( InsightsIAS Mock Test Series for UPSC Preliminary Exam 2020 )

2022. SPAI is a part of India’s PAWS effort. In partnership with 11 other snow leopard range
countries under GSLEP, India will take part in the jointly initiated efforts. The other snow-
leopard range countries are Afghanistan, Bhutan, China, Mongolia, Kazakhstan,
Kyrgyzstan, Nepal, Tajikistan, Pakistan, Russia and Uzbekistan.

Hence, option (b) is correct.

https://indianexpress.com/article/explained/why-india-and-world-are-counting-snow-leopards-and-ho
w-6088190/

https://www.globalsnowleopard.org/

6 With reference to the ‘Thermoplastics’, consider the following statements:


1. Thermoplastic materials have very high melting points.
2. Thermoplastics can be remolded and recycled without negatively affecting the material’s physical
properties..
3. PVC is an example of thermoplastic.

Which of the statements given above is/are correct?


A. 1 only
B. 2 and 3 only
C. 1 and 2 only
D. 1, 2 and 3

Correct Answer : B

Answer Justification :

A thermoplastic is a material, usually a plastic polymer, which becomes soft when heated and
hard when cooled. They have very low melting point. Hence, statement 1 is incorrect.

Thermoplastic materials can be cooled and heated several times without any change in
their chemistry or mechanical properties. When thermoplastics are heated, they melt to a
liquid. They freeze to a glassy state when cooled enough. Hence, statement 2 is correct.

Thermoplastic materials have many features. Some products made from thermoplastic materials are
used for electronic applications. They protect against electrostatic discharge and radio frequency
interference. Thermoplastics are one of the main two types of plastics.

Thermoplastic can be moulded into any shape. Thermoplastics differ from thermosetting
polymers. Thermosets form irreversible chemical bonds during the curing process.

The most produced plastics are:

Polyethylene (PE)

Polypropylene

4
Total Marks : 200
Online Prelims TEST - 13 (SUBJECT WISE)
( InsightsIAS Mock Test Series for UPSC Preliminary Exam 2020 )

Polyvinyl chloride (PVC). Hence, statement 3 is correct.

https://www.financialexpress.com/auto/car-news/garware-polyester-launches-paint-protection-films-i
n-india-key-benefits-explained/1745722/

7 mHariyali app launched for Environment Protection in Government Colonies, launched by

A. Ministry of Information and Broadcasting


B. Ministry of Environment, Forest and Climate Change
C. Ministry of Rural Development
D. Ministry of Housing & Urban Affairs

Correct Answer : D

Answer Justification :

Ministry of Housing & Urban Affairs has launched the mobile app, “mHariyali,”. The app is aimed to
encourage Public engagement in planting trees and other such Green drives. Hence, option (d) is
correct.

People can now upload information/photos of any plantation done by them, which is linked to app
and will be displayed on the website www.epgc.gov.in. The App provides for automatic geo-tagging
of plants. This app will also enable nodal officers to periodically monitor the plantation. The App is
user friendly and works on any android mobile phone.

8 Consider the following statements


1. The project LOcal Treatment of Urban Sewage Streams for Healthy Reuse (LOTUS HR) is an Indo-
Nepal joint project funded by Department of Biotechnology (DBT), Govt of India
2. The project aims to demonstrate a novel holistic waste-water management approach, that will
produce clean water that can be reused for various proposes, while simultaneously recovering
nutrients and energy from the urban waste water, thus converting drain into profitable mines.

Which of the statements given above is/are correct?


A. 1 only
B. 2 only
C. Both 1 and 2
D. Neither 1 nor 2

Correct Answer : B

Answer Justification :

The project LOcal Treatment of Urban Sewage Streams for Healthy Reuse (LOTUS HR) is an Indo-
Netherlands joint project funded by Department of Biotechnology (DBT), Govt of India and
NWO/STW, The Govt of Netherlands. Hence, statement 1 is incorrect.

5
Total Marks : 200
Online Prelims TEST - 13 (SUBJECT WISE)
( InsightsIAS Mock Test Series for UPSC Preliminary Exam 2020 )

The project aims to demonstrate a novel holistic waste-water management approach, that will
produce clean water that can be reused for various proposes, while simultaneously recovering
nutrients and energy from the urban waste water, thus converting drain into profitable mines.
Hence, statement 2 is correct.

9 Consider the following statements:


1. Recently, the annual Ganges river Dolphin census was undertaken by the World Organisation for
Animal Health in collaboration with the Uttar Pradesh Forest Department.
2. Gangetic River dolphin is listed on CITES Appendix-I.

Which of the statements given above is/are correct?


A. 1 only
B. 2 only
C. Both 1 and 2
D. Neither 1 nor 2

Correct Answer : B

Answer Justification :

The Gangetic River dolphin is primarily found in the Ganges and Brahmaputra Rivers and their
tributaries in Bangladesh, India and Nepal.

Recently, the annual Ganges river Dolphin census was undertaken by the World Wide Fund for
Nature- India in collaboration with the Uttar Pradesh Forest Department along about 250 km. long
riverine stretch of Upper Ganga river basin between Hastinapur Wildlife Sanctuary and Narora
Ramsar site. Hence, statement 1 is incorrect.

It is listed on CITES Appendix-I. Hence, statement 2 is correct.

10 Which of the National Park is known for Great Indian Bustard?

A. Bandipur National Park, Karnataka


B. Desert National Park, Rajasthan
C. Murien National Park, Mizoram
D. Vedanthangal National Park, Tamil Nadu

Correct Answer : B

Answer Justification :

Desert National Park, near Jaisalmer and coastal grasslands of the Abdasa and Mandvi talukas
of Kutch District of Gujarat support some populations.

Great Indian bustard, (Ardeotis nigriceps), large bird of the bustard family (Otididae), one of

6
Total Marks : 200
Online Prelims TEST - 13 (SUBJECT WISE)
( InsightsIAS Mock Test Series for UPSC Preliminary Exam 2020 )

the heaviest flying birds in the world. The great Indian bustard inhabits dry grasslands and
scrublands on the Indian subcontinent; its largest populations are found in the Indian state of
Rajasthan. Hence, option (b) is correct.

11 Consider the following statements:


1. The Green Growth Knowledge Platform (GGKP) was established in 2012 by the World Bank and
United Nation Development Program (UNDP).
2. The United Nations Environment Programme Finance Initiative (UNEP FI) was established as a
platform associating the United Nations and the financial sector globally.

Which of the statements given above is/are correct?


A. 1 only
B. 2 only
C. Both 1 and 2
D. Neither 1 nor 2

Correct Answer : B

Answer Justification :

The Green Growth Knowledge Platform (GGKP) is a global network of international organizations
and experts that identifies and addresses major knowledge gaps in green growth theory and
practice.

The GGKP was established in January 2012 by the Global Green Growth Institute, the
Organisation for Economic Co-operation and Development, the United Nations
Environment Programme and the World Bank. Hence, statement 1 is incorrect.

Founded in 1992 in the context of the Earth Summit in Rio, and based in Geneva, Switzerland, the
United Nations Environment Programme Finance Initiative (UNEP FI) was established as a
platform associating the United Nations and the financial sector globally. The need for this
unique United Nations partnership arose from the growing recognition of the links between finance
and Environmental, Social and Governance (ESG) challenges, and the role financial institutions
could play for a more sustainable world. Hence, statement 2 is correct.

12 Which of the following biosphere reserves is/are included in the UNESCO World Network of
Biosphere Reserves (WNBR)?
1. Khangchendzonga Biosphere Reserve
2. Simlipal Biosphere Reserve
3. Cold Desert Biosphere Reserve

Select the correct answer using the code given below


A. 1 and 2 only
B. 1 only
C. 3 only
D. None

7
Total Marks : 200
Online Prelims TEST - 13 (SUBJECT WISE)
( InsightsIAS Mock Test Series for UPSC Preliminary Exam 2020 )

Correct Answer : A

Answer Justification :

13 With reference to the National Wetlands Committee (NWC), consider the following statements:
1. The functions of NWC would be to monitor the implementation of these rules and oversee various
works carried out by the state governments.
2. It would recommend the designation of wetlands of international importance under the Ramsar
convention
3. It would also advise the government in collaboration with the International organizations on various
issues related to wetland conservation and wetland protection.

Which of the statements given above is/are correct?


A. 1 only
B. 2 and 3 only
C. 1 and 3 only
D. 1, 2 and 3

Correct Answer : D

Answer Justification :

The wetlands conservation and management rules, 2017 stipulates for setting up national wetland
authority (NWC) to be headed by the secretary of the ministry of environment, forest and climate

8
Total Marks : 200
Online Prelims TEST - 13 (SUBJECT WISE)
( InsightsIAS Mock Test Series for UPSC Preliminary Exam 2020 )

change. The functions of NWC would be to monitor the implementation of these rules and oversee
various works carried out by the state governments. Hence, statement 1 is correct.

The NWC will also give advice to the union government on various policies and action programs for
wetland conservation, wetland protection and wise use of wetlands. It would recommend the
designation of wetlands of international importance under the Ramsar convention. Hence,
statement 2 is correct.

It would also advise the government in collaboration with the International organizations on various
issues related to wetland conservation and wetland protection etc. Hence, statement 3 is correct.

https://nrcd.nic.in/npca/npca/npca/index_npca.aspx

14 Which of the following species can be found in Shola Grasslands?


1. Tiger
2. Binturong
3. Nilgiri Tahr
4. Lion tailed macaque

Select the correct answer using the code given below


A. 1, 3 and 4 only
B. 2, 3 and 4 only
C. 1, 2, 3 and 4
D. 3 and 4 only

Correct Answer : A

Answer Justification :

Tiger, Nilgiri Tahr, Lion-Tailed Macaque can be found in Shola Forest.

The binturong also known as bearcat, is a viverrid native to South and Southeast Asia. It is
uncommon in much of its range, and has been assessed as Vulnerable on the IUCN Red List
because of a declining population trend that is estimated at more than 30% since the mid-1980s.

Hence, option (a) is correct.

15 With reference to biopesticides, consider the following statements:


1. Bacillus thuringiensis is an example of biopesticide.
2. Biopesticides generally affect only the target pest and closely related organisms.
3. Bio pesticides are usually inherently less toxic than conventional pesticides.

Which of the statements given above is/are correct?


A. 1 only
B. 2 and 3 only
C. 1 and 2 only

9
Total Marks : 200
Online Prelims TEST - 13 (SUBJECT WISE)
( InsightsIAS Mock Test Series for UPSC Preliminary Exam 2020 )

D. 1, 2 and 3

Correct Answer : D

Answer Justification :

Bio pesticides are made of living things, come from living things, or they are found in nature. They
tend to pose fewer risks than conventional chemicals. Very small quantities can be effective and
they tend to break down more quickly, which means less pollution.

Types of bio pesticides:

Microbes - These are tiny organisms like bacteria and fungi. They tend to be more targeted
in their activity than conventional chemicals. For example, a certain fungus might control
certain weeds, and another fungus might control certain insects. The most common microbial
bio pesticide is Bacillus thuringiensis. Hence, statement 1 is correct.

Substances Found in Nature – These include plant materials like corn gluten, garlic oil, and
black pepper. These also some include insect hormones that regulate mating, molting, and
food-finding behaviors. They tend to control pests without killing them.

Plant-Incorporated Protectants (PIPs) – These are the genes and proteins, which are
introduced into plants by genetic engineering. They allow the genetically modified plant to
protect itself from pests, like certain insects or viruses.

Biopesticides are usually inherently less toxic than conventional pesticides. Hence,
statement 3 is correct.

Biopesticides generally affect only the target pest and closely related organisms, in contrast
to broad spectrum, conventional pesticides that may affect organisms as different as birds,
insects and mammals. Hence, statement 2 is correct.

16 Which of the following organizations conducts Swachch Sarvekshan survey?

A. NITI Aayog
B. NSSO
C. Central Pollution Control Board (CPCB)
D. Quality Council of India (QCI)

Correct Answer : D

Answer Justification :

10
Total Marks : 200
Online Prelims TEST - 13 (SUBJECT WISE)
( InsightsIAS Mock Test Series for UPSC Preliminary Exam 2020 )

Swachh Survekshan (Cleanliness survey) is an annual survey of cleanliness, hygiene and


sanitation in cities and towns across India. It was launched as part of the Swachh Bharat Abhiyan,
which aims to make India clean and free of open defecation by 2 October 2019. The first survey was
undertaken in 2016 and covered 73 cities (53 cities with a population of over a million, and all state
capitals); by 2019 the survey had grown to cover 4237 cities and was said to be the largest
cleanliness survey in the world. The surveys are carried out by Quality Council of India.

Hence, option (d) is correct.

17 With reference to Mangroves, consider the following statements:


1. They are usually found in the littoral regions of tropical and subtropical regions.
2. Some of the mangrove species exhibit Viviparity mode of reproduction
3. The mangroves vegetation in the coastal zone of Kerala is very sparse and thin.

Which of the statements given above is/are correct?


A. 1, and 2 only
B. 1 and 3 only
C. 3 only
D. 1, 2 and 3

Correct Answer : D

Answer Justification :

Mangroves are the characteristics littoral plant formation of tropical and subtropical sheltered
coastlines. Mangroves are trees and bushes growing below the high water level of spring tides
which exhibits remarkable capacity for salt water tolerance. Hence, statement 1 is correct.

They are basically evergreen land plants growing on sheltered shores, typical on tidal flats, deltas,
estuaries, bays, creeks and barrier islands.

Mangroves are exhibit Viviparity mode of reproduction. I.e. seeds germinate in the tree itself. This
is an adaptive mechanism to overcome the problem of germination in saline water. Hence,
statement 2 is correct.

The mangroves vegetation in the coastal zone of Kerala is very sparse and thin. Hence, statement
3 is correct.

18 Singphan Wildlife Sanctuary, sometime seen in the news, is located in

A. Assam
B. Manipur
C. Nagaland
D. West Bengal

11
Total Marks : 200
Online Prelims TEST - 13 (SUBJECT WISE)
( InsightsIAS Mock Test Series for UPSC Preliminary Exam 2020 )

Correct Answer : C

Answer Justification :

The Nagaland government has declared the Singphan Wildlife Sanctuary as an elephant reserve
making it the 30th reserve for jumbos in the country.

An important reserve in North Eastern States, Singphan Elephant Reserve has huge tracts of forest,
strategically located in contiguity with the Abhaypur Reserve Forest of Assam which is frequented
by the moving herds of elephants.

Hence, option (c) is correct.

https://www.business-standard.com/article/pti-stories/singphan-wildlife-sanctuary-declared-30th-ele
phant-reserve-of-the-country-118083101296_1.html

19 With reference to Coastal regulation zones, consider the following statements:


1. Coastal regulation zone is includes only the inter-tidal zone
2. The objective of the coastal regulation zone is preservation of coastal ecology and protection of
livelihoods of traditional fisher community.
3. The Shailesh Naik committee is endorsed dilution of regulatory powers held by the central
government in coastal areas.

Which of the statements given above is/are correct?


A. 1 only
B. 2 and 3 only
C. 1 and 3 only
D. 1, 2 and 3

Correct Answer : D

Answer Justification :

As per the govt notification , the coastal land up to 500 m from the high tide line(HTL) and range of
100m along banks of creeks , estuaries , backwater and rivers subject to tidal fluctuations , is called
coastal regulation zone.

Coastal zone along the country has been placed in four categories. Coastal regulation zone is
includes only the inter-tidal zone and land part of the coastal are and doesn’t include the ocean
part. Hence, statement 1 is correct. The notification regulates the set up an expansion of
industries or processing plants, construction activity, and dumping of wastes, mining etc. it doesn’t
impose any restrictions of fishing activities.

Objective of setting up CRZ are: Hence, statement 2 is correct.

• Protection of livelihood of traditional fisher folk communities.

• Preservation of coastal ecology


12
Total Marks : 200
Online Prelims TEST - 13 (SUBJECT WISE)
( InsightsIAS Mock Test Series for UPSC Preliminary Exam 2020 )

• Promotion of economic activity

Shailesh Naik committee: Shailesh Naik committee report was commissioned in june 2014 after
states expressed dissatisfaction regarding the limitations set by the CRZ notification of 2011. The
committee recommended several relaxations in the terms set up by the 2011 notification. it is also
endorsed dilution of regulatory powers held by the central government in coastal areas. The
recommendations have been put forth with the objective of giving a boost to tourism, port
construction and real estate. Hence, statement 3 is correct.

20 With reference to the Wetlands, consider the following statements:


1. India is signatory of Ramsar Convention.
2. The primary responsibility for the management of the wetlands of India is in the hands of Ministry
of Environment and Forest and Climate Change.
3. The latest wetland added in Ramsar list is Nalsarovar bird sanctuary in Gujarat.
4. The country with the greatest area of listed wetlands is United Kingdom.

Which of the statements given above is/are correct?


A. 1 only
B. 2 and 3 only
C. 1, 2 and 3 only
D. 1, 2, 3 and 4 only

Correct Answer : C

Answer Justification :

In India wetlands continue to be seen in isolation and hardly figure in water resources management
and development plans. Wetlands are not defined under any specific administrative jurisdiction .The
primary responsibility for the management of the wetlands of india is in the hands of ministry of
environment and forest. Hence, statement 2 is correct. Though india is signatory of both Ramsar
Convention and the convention of biological diversity. There is no clear cut regulatory framework
for conservation of wetlands. Hence, statement 1 is correct.

India currently has 26 sites designated as Wetlands of International Importance (Ramsar Sites),
with a surface area of 689,131 hectares. Latest wetland added in Ramsar list is Nalsarovar bird
sanctuary in Gujarat. Hence, statement 3 is correct.

The country with highest number of sites is the United Kingdom with 170 and the country with
greatest area of listed wetlands is Bolivia, with over 140,000 Sq km. Hence, statement 4 is
incorrect.

21 Consider the following statements with respect to Sendai Framework for Disaster Risk Reduction
2015-2030:
1. The Sendai framework is a binding agreement with a time frame of 15 years.
2. It recognizes that the state has the primary role to reduce disaster risks
3. It calls for adopting integrated and inclusive institutional measures for preventing vulnerability to

13
Total Marks : 200
Online Prelims TEST - 13 (SUBJECT WISE)
( InsightsIAS Mock Test Series for UPSC Preliminary Exam 2020 )

disaster, increase preparedness for response and recovery and strengthen resilience.

Which of the statements given above is/are correct?


A. 1 and 2 only
B. 2 and 3 only
C. 1 and 3 only
D. 1, 2 and 3

Correct Answer : B

Answer Justification :

Sendai Framework for Disaster Risk Reduction (SFDRR) 2015-30.

SFDRR is an international Treaty that was approved by UN member states in March 2015 at
Third World Conference on Disaster Risk Reduction held in Sendai, Japan. It is voluntary
and non-binding treaty which recognizes that UN member State has primary role to reduce
disaster risk. It has framework for 15-year (2015 to 2030). Hence statement 1 is incorrect

It calls for sharing the responsibility with other stakeholders including local government, the
private sector and other stakeholders. It is successor of the Hyogo Framework for Action
(2005–2015), which had been the most encompassing international accord on disaster risk
reduction.

It sets of common standards, a comprehensive framework with achievable targets, and a


legally-based instrument for disaster risk reduction.

It calls for adopting integrated and inclusive institutional measures for preventing
vulnerability to disaster, increase preparedness for response and recovery and strengthen
resilience. Hence statement 3 is correct

It recognizes that the state has the primary role to reduce disaster risk, but that responsibility
should be shared with other stakeholders including local government, the private sector and
other stake holders. Hence, statement 2 is correct

It aims to achieve the substantial reduction of disaster risk and loses in lives, livelihoods and
health and in the economic, physical, cultural and environmental assets, over the next 15
years.

The framework was adopted at the 3rd UN world conference on disaster risk reduction in
Sendai, Japan, on March 18th, 2015. The Sendai framework for disaster risk reduction
2015-2030 outlines seven clear targets and four priorities for action to new and existing

14
Total Marks : 200
Online Prelims TEST - 13 (SUBJECT WISE)
( InsightsIAS Mock Test Series for UPSC Preliminary Exam 2020 )

reduce disaster risks.

22 First Inter-State translocation of Tiger is happened between which of the following states?

A. Karnataka and Tamil Nadu


B. Madhya Pradesh and Maharashtra
C. Odisha and Madhya Pradesh
D. None

Correct Answer : C

Answer Justification :

First Inter-State translocation of Tiger is happened between Madhya Pradesh and Odisha

Hence, option (c) is correct.

https://indianexpress.com/article/india/first-inter-state-translocation-project-odisha-gets-second-big-
cat-from-madhya-pradesh-5236415/

23 With reference to the measurement of Bio- diversities, consider the following statements:
1. Species evenness is measure of the relative abundance of the different species making up the
richness of an area.
2. Simpson’s index which takes only species richness into account.
3. Gamma diversity is a measurement of the overall diversity for different ecosystems within a region.
4. Alpha diversity is refers to diversity within a particular area and is measured by counting the
numbers of taxa within the ecosystem.

Which of the statements given above is/are correct?


A. 1, 2 and 3 only
B. 1, 3 and 4 only
C. 1, 2 and 4 only
D. 1, 2, 3 and 4

Correct Answer : B

Answer Justification :

Scientists use different biodiversity indices to measure diversity, and no single one will always be
appreciate the two main factors taken in to account when measuring diversity are Richness and
Evenness.

Species richness: it is number of different species represented in an ecological community,


landscape or region.

15
Total Marks : 200
Online Prelims TEST - 13 (SUBJECT WISE)
( InsightsIAS Mock Test Series for UPSC Preliminary Exam 2020 )

Species evenness: evenness is measure of the relative abundance of the different species making
up the richness. Hence, statement 1 is correct.

Many diversity indices have been developed that combine different measures of biodiversity. One is
called Simpson’s index, which takes into account both species richness and an evenness of
abundance among the species present. Hence, statement 2 is incorrect.

Diversity may be measured at different scales. These are 3 indices used by ecologist:

Alpha diversity refers to diversity within a particular area and is measured by counting the
number of taxa within the ecosystem

Beta diversity is measures diversity between ecosystems. Hence, statement 4 is correct.

Gamma diversity is a measurement of the overall diversity for different ecosystems within a
region. Hence, statement 3 is correct.

24 With reference to SYSTEM OF AIR QUALITY AND WEATHER FORECASTING AND RESEARCH
(SAFAR), consider the following statements:
1. It was introduced by the Ministry of Environment, Forest and Climate Change (MoEFCC).
2. It monitors air quality parameters such as PM2.5, PM10, Water vapour, Ozone, Nitrogen Oxides,
Carbon Dioxide, Chlorofloro Carbons.

Which of the statements given above is/are correct?


A. 1 only
B. 2 only
C. Both 1 and 2
D. Neither 1 nor 2

Correct Answer : D

Answer Justification :

The system, first of its kind in the country, was developed indigenously in record time by Indian
Institute of Tropical Meteorology, Pune and operationalized by India Meteorological Department
(IMD). It was introduced by the Ministry of Earth Science. Hence, statement 1 is incorrect.

In addition to regular air quality parameters like PM2.5, PM10, Sulfur Dioxide, Ozone,
Nitrogen Oxides, Carbon Monoxide, the system will also monitor the existence of Benzene,
Toluene and Xylene. Hence, statement 2 is incorrect.

https://pib.gov.in/newsite/printrelease.aspx?relid=180824

25 With reference to NATIONAL DISASTER MANAGEMENT AUTHORITY (NDMA), consider the following statements:

16
Total Marks : 200
Online Prelims TEST - 13 (SUBJECT WISE)
( InsightsIAS Mock Test Series for UPSC Preliminary Exam 2020 )

1. It is a statutory organization.
2. It works under the aegis of Ministry of Home Affairs.
3. The Home Minister is the ex-officio chairperson of NDMA.

Which of the statements given above is/are correct?


A. 1 and 2 only
B. 2 and 3 only
C. 1 only
D. 1, 2 and 3

Correct Answer : A

Answer Justification :

NDMA, established through the Disaster Management Act, 2005 is an agency of the Ministry
of Home Affairs whose primary purpose is to coordinate response to natural or man-made
disasters and for capacity-building in disaster resiliency and crisis response. Hence, both
statement 1 and statement 2 are correct.

The Prime Minister is the ex-officio chairperson of NDMA. Hence, statement 3 is


incorrect.

26 Which one of the following industries is the maximum consumer of water in India?

A. Pulp & Paper


B. Jute textile
C. Thermal Power
D. Iron and Steel

Correct Answer : C

Answer Justification :

Of the total water use by the industry, thermal power plants are the biggest users of water
and account for 88% of the total industrial water use. They are followed by engineering (5.05%)
pulp & paper (2.26%) and textiles (2.07%) industries.

Hence, option (c) is correct.

27 Which of the following organization publishes the ‘Global biodiversity Outlook’?

A. Biodiversity International
B. Convention on Biological Diversity (CBD)
C. United Nations Environment Programme (UNEP)
D. Food and Agriculture Organization (FAO)

17
Total Marks : 200
Online Prelims TEST - 13 (SUBJECT WISE)
( InsightsIAS Mock Test Series for UPSC Preliminary Exam 2020 )

Correct Answer : B

Answer Justification :

Global Biodiversity Outlook (GBO) is the flagship publication of the Convention on


Biological Diversity. It is a periodic report that summarizes the latest data on the status and
trends of biodiversity and draws conclusions relevant to the further implementation of the
Convention.

Hence, option (b) is correct.

https://www.cbd.int/gbo/

28 Which of the following statements is/are true about the ‘Wildlife Crime Control Bureau’?
1. It is a statutory body established under Wildlife (Protection) Act, 1972
2. It establishes a centralized wildlife crime data bank.

Select the correct answer using the code given below:


A. 1 only
B. 2 only
C. Both 1 and 2
D. Neither 1 nor 2

Correct Answer : C

Answer Justification :

Wildlife Crime Control Bureau is a statutory multi-disciplinary body established by the


Government of India under the Ministry of Environment and Forests, to combat organized wildlife
crime in the country. The Bureau has its headquarter in New Delhi and five regional offices at Delhi,
Kolkata, Mumbai, Chennai and Jabalpur; three sub-regional offices at Guwahati, Amritsar and
Cochin; and five border units at Ramanathapuram, Gorakhpur, Motihari, Nathula and Moreh.

Under Section 38 (Z) of the Wildlife (Protection) Act, 1972, it is mandated to collect and
collate intelligence related to organized wildlife crime activities and to disseminate the same to
State and other enforcement agencies for immediate action so as to apprehend the criminals;
Hence, statement 1 is correct.

To establish a centralized wildlife crime data bank; co-ordinate actions by various agencies in
connection with the enforcement of the provisions of the Act; Hence, statement 2 is correct.

Assist foreign authorities and international organization concerned to facilitate co-


ordination and universal action for wildlife crime control;

capacity building of the wildlife crime enforcement agencies for scientific and professional
investigation into wildlife crimes and assist State Governments to ensure success in prosecutions

18
Total Marks : 200
Online Prelims TEST - 13 (SUBJECT WISE)
( InsightsIAS Mock Test Series for UPSC Preliminary Exam 2020 )

related to wildlife crimes;

Advise the Government of India on issues relating to wildlife crimes having national and
international ramifications, relevant policy and laws. It also assists and advises the Customs
authorities in inspection of the consignments of flora & fauna as per the provisions of Wild Life
Protection Act, CITES and EXIM Policy governing such an item.

http://wccb.gov.in/Content/AboutUs.aspx

29 With reference to the Convention on Conservation of Migratory Species of Wild Animals (CMS), the
Bonn convention, consider the following statements:
1. It is an environmental treaty under the aegis of the International Union for Conservation of Nature
(IUCN)
2. It is the only global convention specializing in the conservation of migratory species.
3. The Conference of the Parties (COP) is the decision-making body of the Convention.

Which of the statements given above is/are correct?


A. 1 and 2 only
B. 1 and 3 only
C. 2 and 3 only
D. 1, 2 and 3

Correct Answer : C

Answer Justification :

As an environmental treaty under the aegis of the United Nations Environment Programme,
CMS provides a global platform for the conservation and sustainable use of migratory animals and
their habitats. CMS brings together the States through which migratory animals pass, the
Range States, and lays the legal foundation for internationally coordinated conservation measures
throughout a migratory range. Hence, statement 1 is incorrect.

As the only global convention specializing in the conservation of migratory species, their
habitats and migration routes, CMS complements and co-operates with a number of other
international organizations, NGOs and partners in the media as well as in the corporate sector.
Hence, statement 2 is correct.

The Conference of the Parties (COP) is the decision-making body of the Convention. It
meets at three-yearly intervals. Hence, statement 3 is correct.

Its Standing Committee (StC) gives policy and administrative guidance between meetings.

The Scientific Council (ScC) meets between COP sessions to offer scientific advice and identify
research and conservation priorities.

The Secretariat develops and promotes Agreements, services meetings, supports and supervises
research and conservation projects and co-operates with governments and partner organizations.

19
Total Marks : 200
Online Prelims TEST - 13 (SUBJECT WISE)
( InsightsIAS Mock Test Series for UPSC Preliminary Exam 2020 )

The Secretariat is provided by the United Nations Environment Programme (UNEP) and is based in
Bonn, Germany and has an out-posted office in Abu Dhabi, United Arab Emirates.

https://www.cms.int/en/legalinstrument/cms

30 Which of the following species is/ are considered critically endangered species in India?
1. Malabar Civet
2. Pygmy Hog
3. Sloth bear

Select the correct answer using the code given below:


A. 1 and 2 only
B. 2 only
C. 1 and 3 only
D. 1, 2 and 3

Correct Answer : A

Answer Justification :

The Malabar Civet (Viverra civettina) is considered to be one of the world’s rarest
mammals. It is endemic to India and was first reported from Travancore, Kerala. It is nocturnal in
nature and found exclusively in the Western Ghats.

Habitat: Wooded plains and hill slopes of evergreen rainforests.

Distribution: Western Ghats.

Threats: Deforestation and commercial plantations are major threats.

The Pygmy Hog (Porcula salvania) is the world’s smallest wild pig, with adults weighing only 8
kgs. This species constructs a nest throughout the year. It is one of the most useful indicators of the
management status of grassland habitats.

The grasslands where the pygmy hog resides are crucial for the survival of other endangered
species such as Indian Rhinoceros (Rhinoceros unicornis), Swamp Deer (Cervus duvauceli), Wild
Buffalo (Bubalus arnee), Hispid Hare (Caprolagus hispidus), Bengal Florican (Eupodotis
bengalensis) and Swamp Francolin (Francolinus gularis).

Sloth Bear is a vulnerable species not critically endangered species.

Hence, option (a) is correct.

http://envfor.nic.in/sites/default/files/critically_endangered_booklet.pdf

31 In which one of the following States is the Great Himalayan National Park is located?

20
Total Marks : 200
Online Prelims TEST - 13 (SUBJECT WISE)
( InsightsIAS Mock Test Series for UPSC Preliminary Exam 2020 )

A. Arunachal Pradesh
B. Himachal Pradesh
C. Sikkim
D. Mizoram

Correct Answer : B

Answer Justification :

The Great Himalayan National Park is one of the most famous national parks of India and
a UNESCO World Heritage Site. Stretched across an area of 754sq. km, it is situated in Kullu
district, Himachal Pradesh. It consists mostly of temperate and alpine forests along with virgin
coniferous forests. The park also covers enormous areas of alpine pastures and glaciers. Several
important wildlife species of Western Himalayas like musk deer, goral, leopard, snow leopard,
serow, bharal, brown bear, kalij, monal, tragopan, cheer, kolkas and snow cock, besides others, can
be found here.

Hence, option (b) is correct.

32 Which of the following statements is/are true regarding ‘Secure Himalaya’ project?
1. The project was launched by the Ministry of Environment, Forests and Climate Chang in
collaboration with United Nations Development Programme (UNDP)
2. The project covers Arunachal Pradesh, Sikkim and Assam.

Which of the statements given above is/are correct?


A. 1 only
B. 2 only
C. Both 1 and 2
D. Neither 1 nor 2

Correct Answer : A

Answer Justification :

The Government of India and United Nations Development Programme, with support from
the Global Environment Facility, are implementing a new programme in the high altitude Himalayas
entitled “SECURE Himalayas - Securing livelihoods, conservation, sustainable use and
restoration of high range Himalayan ecosystems”, to ensure conservation of locally and
globally significant biodiversity, land and forest resources in the high Himalayan ecosystem, while
enhancing the lives and livelihoods of local communities. Hence, statement 1 is correct.

It covers the high Himalayan Ecosystem spread over Uttarakhand, Sikkim, Jammu & Kashmir
and Himachal Pradesh. Hence, statement 2 is incorrect.

http://www.in.undp.org/content/india/en/home/projects/securing-livelihoods-in-the-himalayas.html

21
Total Marks : 200
Online Prelims TEST - 13 (SUBJECT WISE)
( InsightsIAS Mock Test Series for UPSC Preliminary Exam 2020 )

33 Consider the following:


1. Birdlife International
2. International Fund for Animal Welfare (IFAW)
3. Wetlands International

Which of the above organizations related to wildlife protection is/are international non-
governmental Organizations?
A. 1 and 3 only
B. 1 only
C. 2 and 3 only
D. 1, 2 and 3

Correct Answer : D

Answer Justification :

BirdLife International is a global partnership of conservation organisations that strives to


conserve birds, their habitats, and global biodiversity, working with people towards sustainability in
the use of natural resources.

International Fund for Animal Welfare (IFAW) is a global non-profit organization that protects
animals and the places they call home. Founded in 1969, IFAW has provided almost 50 years of
global leadership, innovation and hands-on assistance to animals and communities in need. Our
holistic conservation approach includes reducing the demand and exploitation of wildlife at the
source; working together with communities and enforcers to shut down poaching networks;
pioneering first-ever successes in the rescue, rehabilitation and release of orphan animals back into
the wild; activating where needed most in the wake of global disasters; and advocating through
expert research and solutions to the greatest threats to our planet’s populations and habitats.

https://www.ifaw.org/united-states/about-ifaw

Wetlands International is a global organisation that works to sustain and restore wetlands and
their resources for people and biodiversity. It is an independent, not-for-profit, global
organisation, supported by government and NGO membership from around the world.

Hence, option (d) is correct.

34 With reference to the ‘National Nature Camps Programme’, consider the following statements:
1. It is an initiative of Ministry of Environment, Forest and Climate Change.
2. It is aimed at creating greater awareness, understanding and empathy of children with and for the
environment.

Which of the statements given above is/are correct?


A. 1 only
B. 2 only
C. Both 1 and 2
D. Neither 1 nor 2
22
Total Marks : 200
Online Prelims TEST - 13 (SUBJECT WISE)
( InsightsIAS Mock Test Series for UPSC Preliminary Exam 2020 )

Correct Answer : C

Answer Justification :

The National Nature Camps Programme is an initiative of the Ministry of Environment and
Forests in Environment Education, which is aimed at creating greater awareness,
understanding and empathy of children with and for the environment. Through this initiative
it is hoped that every child who goes through middle school (classes VI to VIII) will get at least one
opportunity for a 2-3 day camping experience during these years.

Hence, both statements are correct.

The ‘Nature experience’ for children and teachers has huge potential to trigger their
sensitivity towards nature appreciation and conservation, leading to positive environmental actions
at different levels.

https://www.wwfindia.org/?11021/the-national-nature-camps-programme---a-true-nature-experience-
for-children

35 Which of the following is/are the characteristics that the pollutant should possess in order for bio
magnification to occur?
1. A pollutant should be short lived.
2. A pollutant should be biologically inactive.
3. A pollutant should not be soluble in fats.

Select the correct answer using the code given below:


A. 1 only
B. 2 only
C. 1, 2 and 3
D. None

Correct Answer : D

Answer Justification :

Biomagnification stands for Biological Magnification, which means the increase of


contaminated substances or toxic chemicals that take place in the food chains. These
substances often arise from intoxicated or contaminated environments.

The contaminants include heavy metals namely mercury, arsenic, pesticides such as DDT,
and polychlorinated biphenyls (PCBs) compounds which are then taken up by organisms
because of the food they consume or the intoxication of their environment.

Pollutant needs to satisfy characteristics like long life, biologically active, soluble in fat etc
to make biomagnification possible.

23
Total Marks : 200
Online Prelims TEST - 13 (SUBJECT WISE)
( InsightsIAS Mock Test Series for UPSC Preliminary Exam 2020 )

Hence, option (d) is correct.

36 Consider the followings statements


1. Only 10 percent of the energy is transferred to each tropic level from the lower tropic level.
2. A given species cannot occupy more than one tropic level in the same ecosystem at same time.

Which of the statements given above is/are correct?


A. 1 only
B. 2 only
C. Both 1 and 2
D. Neither 1 nor 2

Correct Answer : A

Answer Justification :

The number of tropic levels in the grazing food is restricted as the transfer of energy follows 10 per
cent law. Only 10 percent of the energy is transferred to each tropic level from the lower tropic
level. Hence, statement 1 is correct.

A given organism may occupy more than one tropic level simultaneously. One must remember that
the tropic level represents a functional level, not a species as such. A given species may occupy
more than one tropic level in the same ecosystem at same time. For example, a sparrow is a primary
consumer when it eats seeds, fruits, peas , and a secondary consumer when it eats insects and
worms . Hence, statement 2 is incorrect.

37 With reference to the Energy flow in ecosystem, consider the following statements:
1. The energy is always flow from higher to lower tropic level.
2. Detritus food chain goes from dead organic matter into microorganisms and then to organisms
feeding on detritus and their predators.
3. The amount of energy increase at successive tropic levels.

Which of the statements given above is/are not correct?


A. 1 and 2 only
B. 2 and 3 only
C. 1 and 3 only
D. 1, 2 and 3

Correct Answer : C

Answer Justification :

Energy is basic force responsible for all metabolic activities. The flow of energy from producers to
top consumers called is energy flow.

24
Total Marks : 200
Online Prelims TEST - 13 (SUBJECT WISE)
( InsightsIAS Mock Test Series for UPSC Preliminary Exam 2020 )

Energy flows through the tropic levels from producers to subsequent tropic levels. This energy is
always flow from lower (producers) to higher tropic (consumer) level. Hence, statement 1 is
incorrect. It never flows in the reverse direction that is from carnivores to herbivores to producers.

Detritus food chain goes from dead organic matter into microorganisms and then to organisms
feeding on detritus and their predators. Such ecosystems are thus less dependent on direct solar
energy. Hence, statement 2 is correct.

The energy is passed through the system in food chains and webs. The flow of energy is
unidirectional. The important point is note that the amount of energy decreases at successive tropic
levels. Hence, statement 3 is incorrect.

38 With reference to the “Nutrient Cycles”, consider the following statements:


1. The hydrological cycle is the circulation of water within the earth’s hydrosphere in different forms.
2. There is an inexhaustible supply of nitrogen in the atmosphere and elemental form can be used
directly by the all living organisms.
3. Carbon cycle is mainly the conversion of carbon dioxide and some of the carbohydrates are directly
used by plants itself.

Which of the statements given above is/are correct?


A. 1 and 2 only
B. 1 and 3 only
C. 2 and 3 only
D. 1, 2 and 3

Correct Answer : B

Answer Justification :

The flow of energy is one way. In contrast, the nutrients which are needed to produce organic
material are circulated round the system and are reused several times. All natural elements are
capable of being absorbed by plants usually as gases from the air or soluble salt form the soil but
only oxygen, carbon, hydrogen, nitrogen are needed in large quantities. These substances are
known as macronutrients on the basis of fats, carbohydrates, and proteins.

Water undergoes a cycle from the ocean to land and; and to ocean. The hydrological cycle is the
circulation of water within the earth’s hydrosphere in different forms. Hence, statement 1 is
correct.

Nitrogen cycle is an example of gaseous type. It is probably the most complete of nutrient cycles.
There is an exhaustible supply of nitrogen in atmosphere but the elemental form cannot be used
directly by most of living organisms. Hence, statement 2 is incorrect.

The carbon cycle is mainly conversion of carbon dioxide. This conversion is initiated by the fixation
of carbon dioxide from the atmosphere through photosynthesis. Some of carbohydrates are utilized
directly by the plants itself. Hence, statement 3 is correct.

25
Total Marks : 200
Online Prelims TEST - 13 (SUBJECT WISE)
( InsightsIAS Mock Test Series for UPSC Preliminary Exam 2020 )

39 Global Soil Biodiversity Atlas, sometime seen in the news, is prepared by

A. United Nations Convention to Combat Desertification (UNCCD)


B. United Nations Environment Programme (UNEP)
C. Bioversity International
D. World Wide Fund for Nature

Correct Answer : D

Answer Justification :

https://www.thehindu.com/sci-tech/energy-and-environment/wwf-sees-grave-danger-to-indias-soil-bi
odiversity/article25372875.ece

40 With reference to the Oceanic Aquatic Biomes, consider the following statements:
1. The euphotic zone of ocean floor receives good amount of sunlight.
2. Planktons are actively moving organisms with well-developed locomotory organs.
3. Nektons are passively drifting on floating organisms.

Which of the statements given above is/are correct?


A. 1 and 2 only
B. 1 only
C. 1 and 3 only
D. 1, 2 and 3

Correct Answer : B

Answer Justification :

The aquatic biome includes the habitats around the world that are dominated by water—from
tropical reefs to brackish mangroves, to Arctic lakes. Aquatic biomes are of four types: oceanic,
lakes and ponds, marshes, streams–rivers.

Oceanic biomes has high concentration of salts with an average of 3.5%. The most abundant ions
are sodium and chloride. Vertical zones occurs I ocean depending on the availability of light for
photosynthesis. The upper 200 m layer of ocean floor receives good amount of sunlight. it is called
euphotic zone . Hence, statement 1 is correct. Some light penetrates deeper though it is
insufficient for photosynthesis. It is depth between 200-2000 m and is called aphotic zone.

Oceanic life can be grouped into following categories.

Plankton: these are passively drifting on floating organisms. They are phytoplankton and zoo
plankton. Hence, statement 2 is incorrect.

26
Total Marks : 200
Online Prelims TEST - 13 (SUBJECT WISE)
( InsightsIAS Mock Test Series for UPSC Preliminary Exam 2020 )

Nektons: These are actively moving organisms with well-developed locomotory organs.
Hence, statement 3 is incorrect.

41 With reference to the Coral Reefs, consider the following statements:


1. Only hard corals build reefs.
2. The tissues of corals themselves are actually not the beautiful colors of the coral reefs, but are
instead clear.
3. Coral reefs are occur in shallow tropical areas where the sea water is clean, clear and warm.
4. Reef building corals are symbolic association of Polyps and Zooxanthellae.

Which of the statements given above is/are correct?


A. 1 and 2 only
B. 2 and 3 only
C. 1, 2 and 4 only
D. 1, 2, 3 and 4

Correct Answer : D

Answer Justification :

Coral is a living animal. Coral has a symbiotic relationship with zooxanthellae microscopic algae
which lives on corals. zooxanthellae assist the coral in nutrient production through its
photosynthetic activities . These activities provide the corals with fixed carbon compounds for
energy, enhance classification, and mediate elemental nutrient flux.

The tissues of corals themselves are actually not the beautiful colors of the coral reefs, but are
instead clear (white). The corals receive their coloration from the zooxanthellae living within their
tissues. Hence, statement 2 is correct.

There are two types of corals: hard orals and soft corals, such as sea fens and gorgonians. Only hard
corals build reefs. Hence, statement 1 is correct.

The reef building corals are symbolic association of polyps (coral animals) and zooxanthellae ( the
microscopic algae ) . Hence, statement 4 is correct. Corals are occur in shallow tropical areas
where the sea water is clean, clear and warm. Hence, statement 3 is correct.

42 Consider the following statements:


1. Lichen is a symbiotic partnership of two separate organisms, a fungus and an alga.
2. Lichen is an example of indicator species.

Which of the above statements is/are correct?


A. 1 only
B. 2 only
C. Both 1 and 2
D. Neither 1 nor 2

27
Total Marks : 200
Online Prelims TEST - 13 (SUBJECT WISE)
( InsightsIAS Mock Test Series for UPSC Preliminary Exam 2020 )

Correct Answer : C

Answer Justification :

A lichen is a composite organism that arises from algae or cyanobacteria living among
filaments of multiple fungi species in a mutualistic relationship. The combined lichen has
properties different from those of its component organisms. Lichens come in many colors, sizes, and
forms. Hence, statement 1 is correct.

Indicator species, organism—often a microorganism or a plant—that serves as a measure of the


environmental conditions that exist in a given locale. Lichens are examples of indicator species.
Hence, statement 2 is correct.

43 Which of the following missions is/are adopted under the National action plan on climate change
(NAPCC)?
1. National Solar Mission
2. National Mission for Green India
3. National Mission for Rural Development
4. National Soil mission

Select the correct answer using the code given below:


A. 1 and 2 only
B. 1, 2 and 4 only
C. 1, 3 and 4 only
D. 1, 2, 3 and 4

Correct Answer : A

Answer Justification :

The National Action Plan on Climate Change (NAPCC) is a govt document that includes eight
ambitious goals set for the country to achieve.

The eight missions are:

National Solar Mission

National Mission for Enhanced Energy Efficiency

National Mission on Sustainable Habitat

National Water Mission

28
Total Marks : 200
Online Prelims TEST - 13 (SUBJECT WISE)
( InsightsIAS Mock Test Series for UPSC Preliminary Exam 2020 )

National Mission for Sustaining the Himalayan Ecosystem

National Mission for a Green India

National Mission for Sustainable Agriculture

National Mission on Strategic Knowledge for Climate Change

Hence, option (a) is correct.

44 With reference to the ‘Central Zoo Authority (CZA)’, consider the following statements:
1. It was established under the Wildlife Protection Act, 1972.
2. It is a statutory body under the Prime Minister Office (PMO)

Which of the statements given above is/are correct?


A. 1 only
B. 2 only
C. Both 1 and 2
D. Neither 1 nor 2

Correct Answer : A

Answer Justification :

Central Zoo Authority was established as a statutory body under the Ministry of Environment
& Forests by the Government of India in the year 1992. Hence, statement 2 is incorrect.

The Authority consists of a Chairman, ten members and a Member Secretary. Section 38 A to 38 J
was added to the Wildlife (Protection) Act, 1972 for establishment of the Central Zoo
Authority in India. Hence, statement 1 is correct.

The main objective of this Authority is to complement and strengthen the national effort in
conservation of the rich biodiversity of the country, particularly the fauna as per the National Zoo
Policy, 1998.

Other objectives of this Authority include- enforcing minimum standards and norms for upkeep and
healthcare of animals in Indian zoos and to control mushrooming of unplanned and ill-conceived
zoos.

Every zoo in the country is required to obtain recognition from the Authority for its operation. The
Authority evaluates the zoos with reference to the parameters prescribed under the Rules and
grants recognition, accordingly.

The Authority’s role is more of a facilitator than a regulator. It therefore, provides technical and

29
Total Marks : 200
Online Prelims TEST - 13 (SUBJECT WISE)
( InsightsIAS Mock Test Series for UPSC Preliminary Exam 2020 )

financial assistance to such zoos which have the potential to attain the desired standard in animal
management.

Apart from the primary function of grant of recognition. the Central Zoo Authority also regulates
the exchange of animals of endangered category listed under Schedule- I and II of the Wildlife
(Protection) Act, 1972 among zoos.

http://cza.nic.in/page/en/introduction

45 Which of the following is/are example(s) of Mutualism interaction between the species?
1. Leech and Cattle
2. Rhizobium and Leguminous plants
3. Coral Polyps and Zooxanthellae

Select the correct answer using the code given below:


A. 1 only
B. 2 and 3 only
C. 3 only
D. 1, 2 and 3

Correct Answer : B

Answer Justification :

Mutualism, association between organisms of two different species in which each benefits.
Mutualistic arrangements are most likely to develop between organisms with widely different living
requirements.

Several well-known examples of mutualistic arrangements exist. The partnership between


nitrogen-fixing bacteria and leguminous plants is one example. In addition, cows possess rumen
bacteria that live in the digestive tract and help digest the plants the cow consumes. Associations
between tree roots and certain fungi are often mutualistic

Coral Polyps are also an example of Mutualism.

Leech and Cattle is an example for parasitism.

Hence, option (b) is correct.

46 With reference to Air (Prevention and Control of Pollution) Act, 1981, consider the following
statements.
1. The Act makes provisions for the establishing of Central Pollution Control Board (CPCB) at the apex
level and State Pollution Control Boards at the state level.
2. This act doesn't covers noise pollution.
3. The act say that no person shall, without the previous consent of the State Board, establish or
operate any industrial plant in an air pollution control area.

30
Total Marks : 200
Online Prelims TEST - 13 (SUBJECT WISE)
( InsightsIAS Mock Test Series for UPSC Preliminary Exam 2020 )

Which of the statements given above is/are correct?


A. 2 only
B. 1 and 3 only
C. 2 and 3 only
D. 1, 2 and 3

Correct Answer : B

Answer Justification :

The Air (Prevention and Control of Pollution) Act, 1981 an Act of the Parliament of India to control
and prevent air pollution in India. It was amended in 1987. The Act makes provisions for the
establishing of Central Pollution Control Board (CPCB) at the apex level and State Pollution Control
Boards at the state level. Hence, statement 1 is correct

According to this act, the “air pollutant” means any solid, liquid or gaseous substance (including
noise) present in the atmosphere in such concentration as may be or tend to be injurious to human
beings or other living creatures or plants or property or environment. Hence, statement 2 is
incorrect.

The further provisions of the act say that no person shall, without the previous consent of the State
Board, establish or operate any industrial plant in an air pollution control area. Hence, statement
3 is correct.

Every person to whom consent has been granted by the State Board, shall comply with the
conditions and norms prescribed by the board such as prevention and control of the air pollution.
Failure to do so brings penalty including jail term of at least 1.5 years.

47 Which of the followings are correct with respect to National Afforestation Program.
1. NAP is being operated as a 100% Central Sector Scheme.
2. The NAP Scheme is being implemented through only centralized mechanism.
3. The objective of the scheme is to develop the forest resources with people’s participation, with
focus on improvement in livelihoods of the forest-fringe communities, especially the poor.
4. In India 38% of forest is categorized as subtropical dry deciduous and 30% as tropical moist
deciduous plus other smaller groups.

Which of the statements given above is/are correct?


A. 1 and 2 only
B. 2 and 3 only
C. 1, 3 and 4 only
D. 1, 2, 3 and 4

Correct Answer : C

Answer Justification :

31
Total Marks : 200
Online Prelims TEST - 13 (SUBJECT WISE)
( InsightsIAS Mock Test Series for UPSC Preliminary Exam 2020 )

As per 10th Plan document of Planning Commission relating to the Forests and Environment sector
, the National Afforestation Programme (NAP) Scheme was initiated by scaling-up the SGVSY
project experience and converging all afforestation schemes of the 9th Plan period to avoid duplicity
or redundancy, and at the same time keeping in focus the decentralization agenda of the
government. NAP is being operated as a 100% Central Sector Scheme. Hence, statement 1 is
correct.

The overall objective of the scheme is to develop the forest resources with people’s participation,
with focus on improvement in livelihoods of the forest-fringe communities, especially the poor.
Hence, statement 3 is correct.

NAP Scheme aims to support and accelerate the ongoing process of devolving forest protection,
management and development functions to decentralized institutions of Joint Forest Management
Committee (JFMC) at the village level, and Forest Development Agency (FDA) at the forest division
level.

The NAP Scheme is being implemented through two-tier decentralized mechanism of Forest
Development Agency (FDA) at Forest Division Level and Joint Forest Management Committees
(JFMCs) at the village level. Hence, statement 2 is incorrect.

India has witnessed a minor increase in the percentage of the land area under forest cover from
1950 to 2006. 38% of forest is categorized as subtropical dry deciduous and 30% as tropical moist
deciduous plus other smaller groups. It is taken care that only local species are planted in an area.
Trees bearing fruits are preferred wherever possible due to their function as a food source. Hence,
statement 4 is correct.

48 Which one of the following magazine is published by Birdlife International

A. World Birdwatch
B. Birding world
C. Birders world
D. Wild bird

Correct Answer : A

Answer Justification :

Birdlife International It is a global partnership of conservation organizations that strives to conserve


birds, their habitats and global biodiversity, working with people towards sustainability in the use of
natural resources. It is world’s largest conservation partnerships for conservation of birds and their
habitats. It was earlier known as International Committee for Bird Preservation.

It publishes a quarterly magazine, World Birdwatch, which contains recent news and
authoritative articles about birds, their habitats, and their conservation around the world. It is the
official Red List authority for birds, for the International Union for Conservation of Nature. It also
published important bird areas (IBA) inventory. Hence, option (a) is correct.

32
Total Marks : 200
Online Prelims TEST - 13 (SUBJECT WISE)
( InsightsIAS Mock Test Series for UPSC Preliminary Exam 2020 )

49 Consider the following statements


1. Bharat stage emission standard regulates the output of air pollutants from internal combustion
engine equipment, including motor vehicles.
2. India has been following American emission norms, though with a time lag of five years.
3. Government is aiming to implement introduction of the cleaner Bharat Stage- VI fuel in the country
by 2020.
4. As per the original timeline in the Auto Fuel Policy, BS-IV is to be adopted across the country by
2017.

Which of the statements given above is/are correct?


A. 1 and 2 only
B. 2 and 3 only
C. 1, 3 and 4 only
D. 1, 2, 3 and 4

Correct Answer : C

Answer Justification :

Bharat Stage emission standards, introduced in 2000, are emission standards that have been set up
by the Central government to regulate the output of air pollutants from internal combustion engine
equipment, including motor vehicles. Hence, statement 1 is correct.

The different norms are brought into force in accordance with the timeline and standards set up by
the Central Pollution Control Board which comes under the Ministry of Environment and Forests
and Climate Change.

Bharat stage emission standard regulates the output of air pollutants from internal combustion
engine equipment, including motor vehicles. India has been following European emission norms.
Hence, statement 2 is incorrect.

Though with a time lag of five years, with BS-IV norms currently applicable in 33 cities where the
required grade of fuel is available while the rest of the country follows BS-III standards.
Government is aiming to implement introduction of the cleaner Bharat Stage- VI fuel in the country
by 2020. Hence, statement 3 is correct.

It intends to skip BS-V. The Ministry of Petroleum and Natural Gas has set a deadline of 2017 by
which time all the vehicles in the country must be BS-IV-compliant and 2020 for the rollout of BS-VI
fuel. As per the original timeline in the Auto Fuel Policy, BS-IV is to be adopted across the country
by 2017, BS V by 2020 and BS-VI by 2024. Hence, statement 4 is correct.

50 Consider the following statements


1. Wetlands International is a non-profit organization established in 1937 as ‘International Wildfowl
Inquiry’
2. Its headquarters is situated in India
3. It is an independent, not-for-profit organization, supported by government and NGO membership
from around the world.

33
Total Marks : 200
Online Prelims TEST - 13 (SUBJECT WISE)
( InsightsIAS Mock Test Series for UPSC Preliminary Exam 2020 )

Which of the statements given above is/are correct?


A. 2 only
B. 1 and 3 only
C. 2 and 3 only
D. 1, 2 and 3

Correct Answer : B

Answer Justification :

Wetlands International is a non-profit organization established in 1937 as ‘International Wildfowl


Inquiry’ Hence, statement 1 is correct.

HQ is in Netherlands. Hence, statement 2 is incorrect.

It is an independent, not-for-profit organization, supported by government and NGO membership


from around the world. Hence, statement 3 is correct.

51 Consider the following statements :


1. The Montreux Record is a register of wetland sites on the List of Wetlands of international
importance
2. It is maintained as part of the Ramsar List.
3. At present, there are three Indian sites on the Montreux Record

Which of the statements given above is/are correct?


A. 2 only
B. 1 and 2 only
C. 2 and 3 only
D. 1, 2 and 3

Correct Answer : B

Answer Justification :

The Montreux Record is a register of wetland sites on the List of Wetlands of international
importance where changes in ecological character have occurred, are occurring, or are likely to
occur as a result of technological developments, pollution or other human interference and
therefore in need of priority conservation attention. Hence, statement 1 is correct.

It is maintained as part of the Ramsar List. Hence, statement 2 is correct.

At present, there are two Indian sites Loktak Lake, Manipur and Keoladeo National Park, Rajasthan
on the Montreux Record. Hence, statement 3 is incorrect.

52 With reference to National Wildlife Action Plan (NWAP), consider the following statements

34
Total Marks : 200
Online Prelims TEST - 13 (SUBJECT WISE)
( InsightsIAS Mock Test Series for UPSC Preliminary Exam 2020 )

1. Recently, India unveiled the NWAP for 2017-2031 spelling out the future road map for wildlife
conservation.
2. The plan has adopted the Landscape approach rather than the earlier strategies more concentrated
on national parks and wildlife sanctuaries.
3. It also emphasizes upon preservation of genetic diversity and sustainable utilization of species and
ecosystem.

Which of the statements given above is/are correct?


A. 2 only
B. 1 and 3 only
C. 2 and 3 only
D. 1, 2 and 3

Correct Answer : D

Answer Justification :

Recently, India unveiled the NWAP for 2017-2031 spelling out the future road map for wildlife
conservation. Hence, statement 1 is correct.

The plan has adopted the Landscape approach rather than the earlier strategies more concentrated
on national parks and wildlife sanctuaries. Hence, statement 2 is correct.

Plan also highlights role of private sector in the wildlife protection by ensuring adequate fund flow
from the Corporate Social Responsibility (CSR) fund.

It also emphasizes upon preservation of genetic diversity and sustainable utilization of species and
ecosystem. Hence, statement 3 is correct.

53 With reference to Collaborative Partnership on Forests (CPF), consider the following statements.
1. It is a formal and compulsory interagency partnership on forests that was in response to an
invitation of Economic and Social Council of the UN (ECOSOC).
2. The Partnership is currently comprised of 20 international organisations, institutions and
secretariats.
3. MISSION of CPF is to help enhance the contribution of all types of forests and trees outside forests.

Which of the statements given above is/are correct?


A. 1 and only
B. 1 and 3 only
C. 3 only
D. 1, 2 and 3

Correct Answer : C

Answer Justification :

35
Total Marks : 200
Online Prelims TEST - 13 (SUBJECT WISE)
( InsightsIAS Mock Test Series for UPSC Preliminary Exam 2020 )

Collaborative Partnership on Forests (CPF) is a informal and voluntary interagency partnership on


forests that was in response to an invitation of Economic and Social Council of the UN (ECOSOC).
Hence, statement 1 is incorrect.

The Partnership is currently comprised of 15 international organisations, institutions and


secretariats. Hence, statement 2 is incorrect.

MISSION of CPF is to help enhance the contribution of all types of forests and trees outside forests.
Hence, statement 3 is correct.

54 Consider the following statements:


1. Fall Armyworm (FAW) is an insect that is native to tropical and subtropical regions of the Africa.
2. It was first detected in Central and Western Africa in early 2016

Which of the statements given above is/are correct?


A. 1 only
B. 2 only
C. Both 1 and 2
D. Neither 1 nor 2

Correct Answer : B

Answer Justification :

Fall Armyworm (FAW) (Spodoptera frugiperda) is an insect pest that feeds on more than 80 crop
species, causing damage to economically important cultivated cereals such as maize, rice, sorghum,
and also to legumes as well as vegetable crops and cotton.

It is an insect that is native to tropical and subtropical regions of the Americas. Hence, statement
is incorrect.

FAW was first detected in Central and Western Africa in early 2016 and has quickly spread across
virtually all of Sub-Saharan Africa. Hence, statement 2 is correct.

55 Consider the following statements


1. Red Panda is closely associated with montane forests with dense bamboo thicket.
2. It mainly is found in Sikkim, West Bengal and Arunachal Pradesh.
3. It has been classified by IUCN as critically Endangered.

Which of the statements given above is/are correct?


A. 2 only
B. 1 and 2 only
C. 2 and 3 only
D. 1, 2 and 3

36
Total Marks : 200
Online Prelims TEST - 13 (SUBJECT WISE)
( InsightsIAS Mock Test Series for UPSC Preliminary Exam 2020 )

Correct Answer : B

Answer Justification :

Red Panda is closely associated with montane forests with dense bamboo-thicket. It is found Sikkim,
West Bengal and Arunachal Pradesh. It is poached for its meat, and for use in medicines, and as a
pet. Hence, statement 1 and 2 are correct.

The IUCN has categorized Red Panda as 'Endangered'. Hence, statement 3 is incorrect.

56 Consider the following statements:


1. NABARD acts as National Implementing Entity (NIE) for National Adaptation Fund for Climate
Change (NAFCC).
2. National water Development Agency (NWDA) is a registered society under the Ministry of Water
Resources, River Development and Ganga Rejuvenation.

Which of the statements given above is/are correct?


A. 1 only
B. 2 only
C. Both 1 and 2
D. Neither 1 nor 2

Correct Answer : C

Answer Justification :

NAFCC fund is meant to assist national and state level activities to meet the cost of adaptation
measures in areas that are particularly vulnerable to the adverse effects of climate change. This
scheme has been taken as Central Sector Scheme with the National Bank for Agriculture
and Rural Development (NABARD) as National Implementing Entity (NIE). Hence,
statement 1 is correct.

NWDA is a registered society under the Ministry of Water Resources, River Development
and Ganga Rejuvenation set up in the year 1982 to carry out detailed studies, surveys and
investigations in respect of Peninsular component of National Perspective for Water Resources
Development. Hence, statement 2 is correct.

57 Consider the following statements:


1. Land Degradation Neutrality (LDN) Fund is managed by a private sector investment management
firm.
2. Earth Overshoot Day is calculated by Global Footprint Network and World Wide Fund for Nature
(WWF).

Which of the statements given above is/are correct?


A. 1 only
B. 2 only

37
Total Marks : 200
Online Prelims TEST - 13 (SUBJECT WISE)
( InsightsIAS Mock Test Series for UPSC Preliminary Exam 2020 )

C. Both 1 and 2
D. Neither 1 nor 2

Correct Answer : C

Answer Justification :

The Land Degradation Neutrality (LDN) Fund is an impact investment fund created to invest in
projects that reduce or reverse land degradation. Mirova, a subsidiary of Natixis Investment
Managers, structured the fund and will manage it on behalf of the United Nations
Convention to Combat Desertification (UNCCD). Hence, statement 1 is correct.

Earth Overshoot Day marks the date when humanity's demand for ecological resources and
services in a given year exceeds what Earth can regenerate in that year. Earth Overshoot
Day is calculated by Global Footprint Network and World Wide Fund for Nature (WWF). Hence,
statement 2 is correct.

58 Consider the following statements:


1. Breathe Life Campaign is a joint campaign led by WHO, UNEP and the Climate and Clean Air
Coalition (CCAC).
2. Breathe India is 15 point action plan proposed by Central Pollution Control Board (CPCB).

Which of the statements given above is/are correct?


A. 1 only
B. 2 only
C. Both 1 and 2
D. Neither 1 nor 2

Correct Answer : A

Answer Justification :

Breathe Life Campaign is a joint campaign led by WHO, UNEP and the Climate and Clean Air
Coalition (CCAC). It aims to mobilise cities and individuals to protect our health and our planet
from the effects of air pollution

Hence, statement 1 is correct.

Niti Aayog has proposed 15-point action plan titled 'Breathe India' for combating air
pollution in ten most polluted cities in the country. Hence, statement 2 is incorrect.

59 Consider the following statements


1. Indian rhino vision 2020 is being implemented by the Department of Environment and Forest
Assam, in partnership with Bodo Autonomous Council.
2. IRV 2020 aims to increase the total rhino population in Assam to 4000 by 2020.

38
Total Marks : 200
Online Prelims TEST - 13 (SUBJECT WISE)
( InsightsIAS Mock Test Series for UPSC Preliminary Exam 2020 )

Which of the statements given above is/are correct?


A. 1 only
B. 2 only
C. Both 1 and 2
D. Neither 1 nor 2

Correct Answer : A

Answer Justification :

Indian rhino vision 2020 is being implemented by the Department of Environment and Forest
Assam, in partnership with Bodo Autonomous Council. It is supported by WWF India, WWF areas
(Asian Rhino and Elephant action strategy) program, the international rhino foundation (IRF), US
fish and wildlife service, save the rhino s campaign of zoological institutions worldwide, and several
NGOs and local conservation groups. Hence, statement 1 is correct.

IRV 2020 aims to increase the total rhino population in Assam to 3000 by 2020. The population of
rhinos are to be distributed in various rhino habit in at least 7 protected areas in order to provide a
long-term sustainable population of one-horned Indian rhino species. Hence, statement 2 is
incorrect.

60 With reference to greater one-horned rhinoceros, consider the following statements


1. The greater one-horned rhinoceros is the largest of the three Asian rhinos and, together with
African white rhinos, is the largest of all rhino species.
2. It is listed as Vulnerable on the IUCN Red List
3. India’s Kaziranga National Park remains the key reserve for this species.

Which of the statements given above is/are correct?


A. 1 only
B. 1 and 3 only
C. 2 and 3 only
D. 1, 2 and 3

Correct Answer : D

Answer Justification :

The greater one-horned rhinoceros is the largest of the three Asian rhinos and, together with
African white rhinos, is the largest of all rhino species. Hence, statement 1 is correct. It is listed
as Vulnerable on the IUCN Red List. Hence, statement 2 is correct. With at least half of the total
population, India’s Kaziranga National Park remains the key reserve for this species. Hence,
statement 3 is correct.

61 Consider the following statements


1. Orangutans are native animals of India.

39
Total Marks : 200
Online Prelims TEST - 13 (SUBJECT WISE)
( InsightsIAS Mock Test Series for UPSC Preliminary Exam 2020 )

2. Orangutans do migrate during winter seasons to warmer regions.

Which of the statements given above is/are correct?


A. 1 only
B. 2 only
C. Both 1 and 2
D. Neither 1 and 2

Correct Answer : D

Answer Justification :

Orangutans are species of great apes native to Indonesia and Malaysia and not native to India.
Hence, statement 1 is incorrect.

Orangutans are not migratory animals and stay rooted in their area, they suffer more due to
deforestation. Hence, statement 2 is incorrect.

Orangutans, with distinctive red fur, are the largest arboreal mammal, spending most of their time
in trees.

62 With reference to Nilgiri Tahr, Consider the following statements


1. It is state animal of Kerala.
2. It is endemic to Western Ghats from the Nilgiris to Kanyakumari.
3. The larger concentration is in Periyar National Park

Which of the statements given above is/are correct?


A. 2 and 3 only
B. 1 and 3 only
C. 2 only
D. 1, 2 and 3

Correct Answer : C

Answer Justification :

The Nilgiri tahr known locally as the Nilgiri ibex or simply ibex, is an ungulate that is endemic to
the Nilgiri Hills and the southern portion of the Western Ghats. Hence, statement 2 is correct.

It is state animal of Tamil Nadu. Hence, statement 1 is incorrect.

The state animal of Kerala is Asian Elephant.

The larger concentration is in Eravikulam National Park. Hence, statement 3 is incorrect.

63 Consider the following statements

40
Total Marks : 200
Online Prelims TEST - 13 (SUBJECT WISE)
( InsightsIAS Mock Test Series for UPSC Preliminary Exam 2020 )

1. Particularly Vulnerable Tribal Group (PVTG) are most vulnerable among the tribal groups.
2. 85 tribal groups have been categorized by the Ministry of Home Affairs as Particularly Vulnerable
Tribal Groups (PVTG)’s.
3. Among the listed PVTG’s the highest number are found in Andhra Pradesh.

Which of the statements given above is/are correct?


A. 1 only
B. 1 and 3 only
C. 2 and 3 only
D. 1, 2 and 3

Correct Answer : A

Answer Justification :

Particularly vulnerable tribal group (PVTG) (earlier: Primitive tribal group) is a government of India
classification created with the purpose of enabling improvement in the conditions of certain
communities with particularly low development indices. Particularly Vulnerable Tribal Group
(PVTG) are more vulnerable among the tribal groups. Hence, statement 1 is correct.

75 tribal groups have been categorized by the Ministry of Home Affairs as Particularly Vulnerable
Tribal Groups (PVTG)’s. Hence, statement 2 is incorrect.

Among the 75 listed PVTG’s the highest number are found in Odisha (13), followed by Andhra
Pradesh (12). Hence, statement 3 is incorrect.

64 With reference to the TRAFFIC, consider the following statements


1. It is a joint program of WWF and CITES
2. It closely works with national and state governments and various agencies to curb illegal wildlife
trade.
3. It aims to prohibit the trade in wild plants and animals across the world.

Which of the statements given above is/are correct?


A. 2 only
B. 1 and 2 only
C. 2 and 3 only
D. 1, 2 and 3

Correct Answer : A

Answer Justification :

The TRAFFIC, the Wildlife Trade Monitoring Network, is a leading non-governmental organisation
working on wildlife trade in the context of both biodiversity conservation and sustainable
development.

41
Total Marks : 200
Online Prelims TEST - 13 (SUBJECT WISE)
( InsightsIAS Mock Test Series for UPSC Preliminary Exam 2020 )

It is a joint program of World Wildlife Fund (WWF) and the International Union for
Conservation of Nature (IUCN). It was established in 1976 and has developed into a global
network, research-driven and action-oriented, committed to delivering innovative and practical
conservation solutions.

It closely works with national and state governments and various agencies to curb illegal
wildlife trade.

It aims to ensure that trade in wild plants and animals is not a threat to the conservation
of nature.

https://www.traffic.org/about-us/our-organisation/

65 With reference to the Central Pollution Control Board (CPCB), consider the following statements:
1. It was established in 1974 under the Environment Protection Act (EPA), 1986
2. It co-ordinates the activities of the state pollution control Boards and resolves disputes among them.
3. It works with industries and all levels of government in a wide variety of voluntary pollution
prevention programs and energy conservation efforts.

Which of the statements given above is/are correct?


A. 1 and 3 only
B. 2 only
C. 2 and 3 only
D. 3 only

Correct Answer : C

Answer Justification :

The Central Pollution Control Board (CPCB) of India is a statutory organisation under the Ministry
of Environment, Forest and Climate Change. It was established in 1974 under the Water
(Prevention and Control of pollution) Act, 1974.

It serves as a field formation and also provides technical services to the Ministry of Environment
and Forests under the provisions of the Environment (Protection) Act, 1986. It co-ordinates the
activities of the State Pollution Control Boards by providing technical assistance and
guidance and also resolves disputes among them.

It was established in 1974 under the Water (Prevention and Control of pollution) Act, 1974. The
CPCB is also entrusted with the powers and functions under the Air (Prevention and Control of
Pollution) Act, 1981.

It is responsible for maintaining national standards under a variety of environmental laws, in


consultation with zonal offices, tribal, and local governments. It has responsibilities to conduct
monitoring of water and air quality, and maintains monitoring data. The agency also works with
industries and all levels of government in a wide variety of voluntary pollution prevention
programs and energy conservation efforts.

42
Total Marks : 200
Online Prelims TEST - 13 (SUBJECT WISE)
( InsightsIAS Mock Test Series for UPSC Preliminary Exam 2020 )

https://en.wikipedia.org/wiki/Central_Pollution_Control_Board

66 Consider the following statements


1. The central government has the power to notify minor minerals in India
2. The power to frame policy and legislation relating to minor minerals is entirely delegated to the
State Governments

Which of the statements given above is/are correct?


A. 1 only
B. 2 only
C. Both 1 and 2
D. Neither 1 nor 2

Correct Answer : C

Answer Justification :

In India, the minerals are classified as minor minerals and major minerals.

According to section 3(e) of the Mines and Minerals (Development and Regulation) Act, 1957
“Minor Minerals” means building stones, gravel, ordinary clay, ordinary sand other than sand used
for prescribed purposes, and any other mineral which the Central Government may, by notification
in the Official Gazette, declare to be a minor mineral. (For the purposes of this Act, the word
"minerals” includes all minerals except mineral oils- natural gas and petroleum).

The central government has the power to notify “minor minerals” under section 3 (e) of
the MMDR Act, 1957. On the other hand, as per Section 15 of the MMDR Act, 1957 State
Governments have complete powers for making Rules for grant of concessions in respect of
extraction of minor minerals and levy and collection of royalty on minor minerals.

The power to frame policy and legislation relating to minor minerals is entirely delegated
to the State Governments while policy and legislation relating to the major minerals are
dealt by the Ministry of Mines under Union /Central Government.

http://www.arthapedia.in/index.php?title=Minor_Minerals

67 Consider the following statements


1. Compact Fluorescent Lamp and other mercury containing lamp brought under the purview of rules.
2. Pan India Extended Producer Responsibility authorization by Central Pollution Control Board.
3. The manufacturer is also now responsible to collect e - waste generated during the manufacture of
any electrical and electronic equipment

Which of the statements given above is/are the part of E-Waste Management Rules, 2016
A. 1 only
B. 1, 2 and 3
C. 3 only

43
Total Marks : 200
Online Prelims TEST - 13 (SUBJECT WISE)
( InsightsIAS Mock Test Series for UPSC Preliminary Exam 2020 )

D. None

Correct Answer : B

Answer Justification :

E-Waste (Management) Rules, 2016

Manufacturer, dealer, refurbisher and Producer Responsibility Organization (PRO) have been
introduced as additional stakeholders in the rules.

The applicability of the rules has been extended to components, consumables, spares and
parts of EEE in addition to equipment as listed in Schedule I.

Compact Fluorescent Lamp (CFL) and other mercury containing lamp brought under
the purview of rules.

Collection mechanism based approach has been adopted to include collection centre,
collection point, take back system etc for collection of e - waste by Producers under Extended
Producer Responsibility (EPR).

Option has been given for setting up of PRO , e - waste exchange , e - retailer, Deposit Refund
Scheme as additional channel for implementation of EPR by Producers to ensure efficient
channelization of e - waste.

Provision for Pan India EPR Authorization by CPCB has been introduced replacing
the state wise EPR authorization.

Collection and channelisation of e - waste in Extended Producer Responsibility - Authorisation


shall be i n line with the targets prescribed in Schedule III of the Rules. The phase wise
Collection Target for e - waste, which can be either in number or Weight shall be 30% of the
quantity of waste generation as indicated in EPR Plan during first two year of implementation
of rules followed by 40% during third and fourth years, 50% during fifth and sixth years and
70% during seventh year onwards.

Deposit Refund Scheme has been introduced as an additional economic instrument wherein
the producer charges an additional amount as a deposit at the time of sale of the electrical
and electronic equipment and returns it to the consumer along with interest when the end - of
- life electrical and electronic equipment is returned.

The e - waste exchange as an option has been provided in the rules as an independent market

44
Total Marks : 200
Online Prelims TEST - 13 (SUBJECT WISE)
( InsightsIAS Mock Test Series for UPSC Preliminary Exam 2020 )

instrument offering assistance or independent electronic systems offering services for sale
and purchase of e - waste generated from end - of - life electrical and electronic equipment
between agencies or organizations authorised under these rules.

The manufacturer is also now responsible to collect e - waste generated during the
manufacture of any electrical and electronic equipment and channelise it for
recycling or disposal and seek authorization from SPCB.

The dealer, if has been given the responsibility of collection on behalf of the producer, need to
collect the e - waste by providing the consumer a box and channelize it to Producer.

Dealer or retailer or e - retailer shall refund the amount as per take back system or De posit Refund
Scheme of the producer to the depositor of e - waste.

http://vikaspedia.in/energy/environment/waste-management/e-waste-management/e-waste-manage
ment-rules-2016

68 Consider the following statements


1. Increase minimum thickness of plastic carry bags from 400 to 500 microns
2. Expansion of the jurisdiction of applicability from the municipal area to rural areas.
3. Collection of plastic waste management fee through pre-registration of the producers and
importers.

Which of the statements given above is/are part of Plastic Waste Management Rules, 2016
A. 1 and 2 only
B. 2 only
C. 2 and 3 only
D. None

Correct Answer : C

Answer Justification :

The Plastic Waste Management Rules, 2016 aim to:

Increase minimum thickness of plastic carry bags from 40 to 50 microns and


stipulate minimum thickness of 50 micron for plastic sheets also to facilitate
collection and recycle of plastic waste

Expand the jurisdiction of applicability from the municipal area to rural areas,
because plastic has reached rural areas also

To bring in the responsibilities of producers and generators, both in plastic waste

45
Total Marks : 200
Online Prelims TEST - 13 (SUBJECT WISE)
( InsightsIAS Mock Test Series for UPSC Preliminary Exam 2020 )

management system and to introduce collect back system of plastic waste by the
producers/brand owners, as per extended producers responsibility

To introduce collection of plastic waste management fee through pre-registration of


the producers, importers of plastic carry bags/multilayered packaging and vendors
selling the same for establishing the waste management system

To promote use of plastic waste for road construction as per Indian Road Congress guidelines
or energy recovery, or waste to oil etc. for gainful utilization of waste and also address the
waste disposal issue; to entrust more responsibility on waste generators, namely payment of
user charge as prescribed by local authority, collection and handing over of waste by the
institutional generator, event organizers.

An eco-friendly product, which is a complete substitute of the plastic in all uses, has not been
found till date. In the absence of a suitable alternative, it is impractical and undesirable to
impose a blanket ban on the use of plastic all over the country. The real challenge is to
improve plastic waste management systems.

http://vikaspedia.in/energy/environment/waste-management/plastic-waste-management-rules-2016

69 With reference to the Climate and Clean Air Coalition (CCAC), consider the following statements
1. It is a voluntary partnership of governments, intergovernmental organisations, businesses, scientific
institutions and civil society organisations.
2. India is a not a member of this coalition

Which of the statements given above is/are correct?


A. 1 only
B. 2 only
C. Both 1 and 2
D. Neither 1 nor 2

Correct Answer : C

Answer Justification :

The Climate and Clean Air Coalition is a voluntary partnership of governments,


intergovernmental organizations, businesses, scientific institutions and civil society
organizations committed to improving air quality and protecting the climate through actions to
reduce short-lived climate pollutants.

Our global network currently includes over 120 state and non-state partners, and hundreds of local
actors carrying out activities across economic sectors.

India is not a member of it.

46
Total Marks : 200
Online Prelims TEST - 13 (SUBJECT WISE)
( InsightsIAS Mock Test Series for UPSC Preliminary Exam 2020 )

https://www.ccacoalition.org/en/content/who-we-are

70 Consider the following


1. Guindy National Park
2. Silent Valley National Park
3. Bandipur National Park

Which of the above national parks is/are located in Western Ghats?


A. 2 and 3 only
B. 2 only
C. 1, 2 and 3
D. 1 and 2 only

Correct Answer : A

Answer Justification :

Guindy National Park is a 2.70 km² protected area of Tamil Nadu, located in Chennai, India, is the
8th-smallest National Park of India and one of the very few national parks situated inside a city.

Silent Valley National Park, is a national park in Kerala, India. It is located in the Nilgiri hills, has
a core area of 89.52 km², which is surrounded by a buffer zone of 148 km².

Bandipur National Park, an 874-sq.-km forested reserve in the southern Indian state of
Karnataka, is known for its small population of tigers.

71 With reference to the India State of Forest Report 2017, consider the following statements
1. Forest Survey of India publishes the report for every two years.
2. India’s forest cover has registered the increase of about five per cent between 2015 and 2017.
3. Much of the increase in the forest cover has been observed in Very Dense Forest (VDF).

Which of the statements given above is/are correct?


A. 3 only
B. 2 and 3 only
C. 1 and 3 only
D. 1, 2 and 3

Correct Answer : C

Answer Justification :

Forest Survey of India publishes the report for every two years. It is publishing report since
1987.

India’s forest cover has registered a marginal increase of about one per cent between 2015

47
Total Marks : 200
Online Prelims TEST - 13 (SUBJECT WISE)
( InsightsIAS Mock Test Series for UPSC Preliminary Exam 2020 )

and 2017. This comes to an increase of 8,021 sq km (about 80.20 million hectare) in the total forest
and 2% rise in total tree cover of the country.

Much of the increase in the forest cover has been observed in Very Dense Forest (VDF).
VDF absorbs maximum carbon dioxide

https://pib.gov.in/newsite/PrintRelease.aspx?relid=176496

72 Consider the following statements


1. Elephant corridors are narrow strips of land that allow elephants to move from one habitat patch to
another.
2. There are 183 identified elephant corridors in India
3. Among state corridors, maximum numbers of them are located in Karnataka.

Which of the statements given above is/are correct?


A. 1 only
B. 1 and 3 only
C. 1 and 2 only
D. 1, 2 and 3

Correct Answer : C

Answer Justification :

Elephant corridors are narrow strips of land that allow elephants to move from one habitat patch to
another. Hence, statement 1 is correct.

There are 183 identified elephant corridors in India. Hence, statement 2 is correct.

Out of this 138 are State Elephant Corridors, 28 Inter-State Elephant Corridors and 17 are
International Elephant Corridors.

Among state corridors, maximum numbers of them are located in Meghalaya. Hence,
statement 3 is incorrect.

73 Consider the following statements


#00000
1. Global warming potential (GWP) is a measure of how much energy the emissions of 1 ton of a GHG
will absorb over a given period of time, relative to the emissions of 1 ton of carbon dioxide (CO2).
2. A GWP is calculated over a specific time interval, commonly 10 to 15 years.

Which of the statements given above is/are correct?


A. 1 only
B. 2 only
C. Both 1 and 2
D. Neither 1 nor 2

48
Total Marks : 200
Online Prelims TEST - 13 (SUBJECT WISE)
( InsightsIAS Mock Test Series for UPSC Preliminary Exam 2020 )

Correct Answer : A

Answer Justification :

Global warming potential (GWP) is a measure of how much energy the emissions of 1 ton of a GHG
will absorb over a given period of time, relative to the emissions of 1 ton of carbon dioxide (CO2).
Hence, statement 1 is correct.

The larger the GWP, the more that a given gas warms the Earth compared to CO2 over that time
period.

A GWP is calculated over a specific time interval, commonly 100 or 500 years. Hence, statement 2
is incorrect.

The GWP depends on factors such as the absorption of infrared radiation by a given species, the
spectral location of its absorbing wavelengths and the atmospheric lifetime of the species.

74 Consider the following statements


1. Stockholm Convention is an International treaty and aims to eliminate or restrict the production and
use of Persistent Organic Pollutants (PoPs).
2. USA is a party to the Stockholm Convention.

Which of the statements given above is/are correct?


A. 1 only
B. 2 only
C. Both 1 and 2
D. Neither 1 nor 2

Correct Answer : A

Answer Justification :

Stockholm Convention on Persistent Organic Pollutants is an international environmental treaty,


signed in 2001 and effective from May 2004, that aims to eliminate or restrict the production and
use of persistent organic pollutants (POPs). Hence, statement 1 is correct.

India is a party to this treaty but US is not a party to this treaty. Hence, statement 2 is incorrect.

75 The Faith for Earth Initiative was launched by

A. Conservation International
B. World Wide Fund for Nature
C. Greenpeace
D. United Nations Environment Programme(UNEP)

49
Total Marks : 200
Online Prelims TEST - 13 (SUBJECT WISE)
( InsightsIAS Mock Test Series for UPSC Preliminary Exam 2020 )

Correct Answer : D

Answer Justification :

The Faith for Earth Initiative intends to bridge the gap between scientists and faith communities,
ensuring that faith leaders have access to up-to-date information on the environment that they can
communicate to their adherents. It was launched in 2017 by UNEP. Hence, option (d) is correct.

It aims to create a high level global “Coalition for Creation” is established to facilitate policy
dialogue on environmental issues that would encourage innovative approaches to finding long-
lasting solutions to environmental challenges.

76 Consider the following statements


1. The ICON satellite will study the Earth’s Ionosphere.
2. The ICON mission is the 39th successful launch and satellite deployment by Pegasus rocket.
3. This mission is operated by the University of California.

Which of the statements given above is/are correct?


A. 1 only
B. 1 and 2 only
C. 1 and 3 only
D. 1, 2 and 3

Correct Answer : D

Answer Justification :

National Aeronautics and Space Administration (NASA) has launched a satellite ICON to
detect dynamic zones of Earth’s Ionosphere. The satellite Ionosphere Connection Explorer (ICON)
was launched from an aircraft over the Atlantic Ocean near the Florida coast.

The ICON satellite will study the Earth’s Ionosphere. It includes various layers of the
uppermost atmosphere where free electrons flow freely. Hence, statement 1 is correct.

The ICON mission is the 39th successful launch and satellite deployment by Pegasus rocket.
Hence, statement 2 is correct.

This mission is operated by the University of California. Hence, statement 3 is correct.

It was originally planned to launch in late 2017 but delayed due to the problems with the
Pegasus XL rocket.

50
Total Marks : 200
Online Prelims TEST - 13 (SUBJECT WISE)
( InsightsIAS Mock Test Series for UPSC Preliminary Exam 2020 )

It is equipped with 780-watt solar arrays to power the instruments.

77 Consider the following statements


1. The Union Minister of Earth Sciences has recently launched the Gagan Enabled Mariner’s
Instrument for Navigation and Information (GEMINI) device.
2. GEMINI is a portable receiver that is linked to ISRO-satellites.
3. It will facilitate satellite-based communication that will be highly useful while dealing with cyclones,
high waves, and tsunamis.

Which of the statements given above is/are correct?


A. 1 only
B. 2 and 3 only
C. 1 and 2 only
D. 1, 2 and 3

Correct Answer : D

Answer Justification :

The Union Minister of Earth Sciences has recently launched the Gagan Enabled Mariner’s
Instrument for Navigation and Information (GEMINI) device. Hence, statement 1 is correct

The device is developed for effective dissemination of emergency information and communication
on Ocean States Forecast and mapping of Potential Fishing Zones (PFZ) to fishermen.

GEMINI is a portable receiver that is linked to ISRO-satellites. With this device, the fishermen
outside the signal range of their phone companies (i.e. 10-12 km) can also access warnings and
alerts, as the device can send signals up to 300 nautical miles. Hence, statement 2 is correct.

It will facilitate satellite-based communication that will be highly useful while dealing with cyclones,
high waves, and tsunamis. Hence, statement 3 is correct.

78 The ‘DHRUV’ programme is launched by

A. Ministry of Women and Child Development


B. NITI Aayog
C. Ministry of Defence
D. Ministry of Human Resource Development

Correct Answer : D

Answer Justification :

Recently, the Union Human Resource Development Minister has launched the Pradhan Mantri
Innovative Learning Programme named ‘DHRUV’. Hence, option (d) is correct.

51
Total Marks : 200
Online Prelims TEST - 13 (SUBJECT WISE)
( InsightsIAS Mock Test Series for UPSC Preliminary Exam 2020 )

The objective of the Pradhan Mantri Innovative Learning Programme is to encourage talented
students to realize their full potential and henceforth, contribute to society.

The programme is named DHRUV after the Pole Star with the same name. Every student selected
under this programme will be called as ‘Dhruv Tara’. The students will thus both shine through
their achievements and light a path for others to follow.

79 Consider the following statements


1. The ASI is the premier organization for the archaeological researches and protection of the cultural
heritage of the country.
2. The prime objection of ASI is to maintain the archaeological sites, ancient monuments and remains
of national importance.
3. It functions under the aegis of the Prime Minister's Office.

Which of the statements given above is/are correct?


A. 1 only
B. 2 and 3 only
C. 1 and 2 only
D. 1, 2 and 3

Correct Answer : C

Answer Justification :
The Archaeological Survey of India (ASI), under the Ministry of Culture, is the premier
organization for the archaeological researches and protection of the cultural heritage of the
nation. Hence, statement 1 and 3 is incorrect

The prime objection of ASI is to maintain the archaeological sites, ancient monuments and
remains of national importance. Hence, statement 2 is correct

It also regulates Antiquities and Art Treasure Act, 1972.

80 Consider the following statements


1. The government has launched Surakshit Matritva Aashwasan to provide free health care facilities to
pregnant women and newborns.
2. Under the scheme, all pregnant women, newborns and mothers up to 6 months of delivery will be
able to avail several free health care services such as four antenatal check-ups and six home-based
newborn care visits.
3. The scheme will enable zero expense access to the identification and management of complications
during and after the pregnancy.

Which of the statements given above is/are correct?


A. 1 and 3 only
B. 1 only
C. 1 and 2 only

52
Total Marks : 200
Online Prelims TEST - 13 (SUBJECT WISE)
( InsightsIAS Mock Test Series for UPSC Preliminary Exam 2020 )

D. 1, 2 and 3

Correct Answer : D

Answer Justification :

The government has launched Surakshit Matritva Aashwasan to provide free health care facilities to
pregnant women and newborns. Hence, statement 1 is correct.

The Surakshit Matritva Aashwasan initiative or SUMAN scheme aims to provide dignified and
quality health care at no cost to every woman and newborn visiting a public health facility.

Under the scheme, all pregnant women, newborns and mothers up to 6 months of delivery will be
able to avail several free health care services such as four antenatal check-ups and six home-based
newborn care visits. Hence, statement 2 is correct.

The scheme will enable zero expense access to the identification and management of complications
during and after the pregnancy. The government will also provide free transport to pregnant women
from home to the health facility and drop back after discharge (minimum 48 hrs). Hence,
statement 3 is correct.

81 With reference to the Cape Town Agreement, consider the following statements:
1. It calls for mandatory safety measures for fishing vessels of 124 meters in length and longer.
2. It was adopted by the International Maritime Organization (IMO).
3. It amends some provisions of the Torremolinos Protocol.

Which of the statements given above is/are correct?


A. 3 only
B. 1 and 2 only
C. 2 and 3 only
D. 1, 2 and 3

Correct Answer : C

Answer Justification :

The International Maritime Organization (IMO) aimed at implementing the provisions of an


international treaty on fishing vessel safety has adopted a agreement on fishing vessel safety, to be
known as the “Cape Town Agreement of 2012 on the Implementation of the Provisions of
the 1993 Protocol relating to the Torremolinos International Convention for the Safety of
Fishing Vessels, 1977”. Hence, statement 2 is correct.

The Cape Town Agreement of 2012 will enter into force 12 months after the date on which not less
than 22 States the aggregate number of whose fishing vessels of 24 m in length and over
operating on the high seas is not less than 3,600 have expressed their consent to be bound by it.
Hence, statement 1 is incorrect.

53
Total Marks : 200
Online Prelims TEST - 13 (SUBJECT WISE)
( InsightsIAS Mock Test Series for UPSC Preliminary Exam 2020 )

The Cape Town Agreement of 2012 updates and amends a number of provisions of the
Torremolinos Protocol. Hence, statement 3 is correct.

https://sdg.iisd.org/news/48-states-commit-to-ratify-cape-town-agreement/

http://www.imo.org/en/MediaCentre/PressBriefings/Pages/44-SFV-conf-ends.aspx

82 With reference to the Hunaar Haats, consider the following statements:


1. It aims to provide employment and opportunities for artisans, craftsmen and culinary experts.
2. It is being organised across the country by the Ministry of Micro, Small and Medium Enterprises.

Which of the statements given above is/are correct?


A. 1 only
B. 2 only
C. Both 1 and 2
D. Neither 1 nor 2

Correct Answer : A

Answer Justification :

Hunar Haats are organised by Ministry of Minority Affairs under USTTAD (Upgrading the Skills
& Training in Traditional Arts/Crafts for Development) scheme. Hence, statement 2 is incorrect.

Hunar Haats have become a successful mission to provide employment and income generation
opportunities with platforms for marketing the products of master artisans, craftsmen and
culinary experts belonging to the minority communities. Hence, statement 1 is correct.

http://www.newsonair.com/News?title=Govt-to-provide-employment-to-lakhs-of-craftsmen-through-
Hunar-Haat-in-next-5-yrs%3A-Naqvi&id=373345

83 Marawah Island sometimes seen in the news is located in?

A. Bay of Bengal
B. Arabian Sea
C. Persian Gulf
D. South China Sea

Correct Answer : C

Answer Justification :

The World’s oldest known natural pearl discovered at a Neolithic site on Marawah Island, off
the coast of Abu Dhabi.

54
Total Marks : 200
Online Prelims TEST - 13 (SUBJECT WISE)
( InsightsIAS Mock Test Series for UPSC Preliminary Exam 2020 )

Marawah is a low-lying island off the coast of the Western Region of the Emirate of Abu Dhabi,
the United Arab Emirates. The island is 15 km north of the Khor al-Bazm along the Abu Dhabi
coastline in the southern Persian Gulf.

http://www.newsonair.com/News?title=World%E2%80%99s-oldest-known-natural-pearl-discovered-
on-Abu-Dhabi-Island&id=373261

84 Consider the following statements:


1. India-U.S.A Defence Technology and Trade Initiative (DTII) a flexible mechanism to make sure that
senior leaders from both countries are engaged consistently to strengthen the opportunities in the
field of defence.
2. In 2018, the US granted to India the designation of Strategic Trade Authority Tier 1 or STA-1.

Which of the statements given above is/are correct?


A. 1 only
B. 2 only
C. Both 1 and 2
D. Neither 1 nor 2

Correct Answer : C

Answer Justification :

Defence Technology and Trade Initiative (DTII) is not a treaty or law, it is a flexible
mechanism to make sure that senior leaders from both countries are engaged consistently to
strengthen the opportunities in the field of defence. Its central aims include strengthening India’s
defence industrial base, exploring new areas of technological development and expanding U.S.-
India business ties. Hence, statement 1 is correct.

In August 2018, the US granted to India the designation of Strategic Trade Authority Tier 1
or STA-1, “providing India with greater supply-chain efficiency by allowing US companies
to export a greater range of dual-use and high-technology items to India under
streamlined processes.” This authorization is the equivalent of NATO allying with Japan, South
Korea and Australia. Hence, statement 2 is correct.

https://indianexpress.com/article/explained/explained-us-india-defense-technology-and-trade-initiativ

55
Total Marks : 200
Online Prelims TEST - 13 (SUBJECT WISE)
( InsightsIAS Mock Test Series for UPSC Preliminary Exam 2020 )

e-dtti-6077915/

85 Bhashan Char Island, recently seen in news is located in?

A. Gulf of Kutch
B. Bay of Bengal
C. Gulf of Oman
D. East China Sea

Correct Answer : B

Answer Justification :

The US has urged Bangladesh to postpone relocation of Rohingyas to the uninhabited island
Bhashan Char. Bhashan Char is located in Bay of Bengal.

Government of Bangladesh has been keen to relocate the Rohingya refugees to the Bhashan char
island where arrangements for relocating 1 lakh people have been made at a cost of 23,000 crores
Taka. It has built houses, cyclone shelters, a three-metre high and 13-km long flood-defence
embankment, food warehouses and other basic facilities on the island.

While the government planned to relocate Rohingyas, environmentalists say that the Bhashan Char
which has emerged less than two decades back from the sea falls in an ecologically fragile area
prone to floods, erosion and cyclone. The issue of forced relocation and lack of mobility on the
island has also been raised by organisations working among the Rohingyas.

Hence, option (b) is correct.

http://www.newsonair.com/News?title=US-urges-Bangladesh-to-postpone-relocation-of-Rohingyas&i
d=373450

86 Consider the following pairs:


Harvest Festival: Associated Community
1. The Mopin festival Soligas
2. Wangala Garos
3. Tusu Todas

Which of the pairs given above is/are correctly matched?


A. 2 only
B. 1 and 3 only
C. 3 only
D. 1, 2 and 3

Correct Answer : A

56
Total Marks : 200
Online Prelims TEST - 13 (SUBJECT WISE)
( InsightsIAS Mock Test Series for UPSC Preliminary Exam 2020 )

Answer Justification :

The Mopin festival is a harvest festival celebrated by the Galo tribe of Arunachal Pradesh
in the Galo months of Lumi and Luki (March and April, according to the Gregorian calendar). The
presiding deity of the festival is Mopin Ane—the goddess of fertility and prosperity of the animist
religion of Donyi-Polo. Hence, pair 1 is incorrectly matched.

Wangala is celebrated by the people of the Garo tribe from Meghalaya, Nagaland and Assam.
Various food items made from rice—a staple in this region—and rice beer (called
chubitchi/chubok/chu)are consumed during the festivities. Hence, pair 2 is correctly matched.

Tusu is a harvest festival of the Kurmi community of West Bengal, Jharkhand and Odisha.
It is celebrated from the last day of Aghrahayan to the last day of the Bengali month of Poush (mid-
November to mid-December).

Hence, pair 3 is correctly matched.

https://www.telegraphindia.com/culture/wangala-tusu-karma-8-lesser-known-harvest-festivals/cid/16
88761

87 With reference to the Aflatoxin, consider the following statements:


1. They are a family of toxins produced by certain bacteria that are found on agricultural crops such as
maize (corn), peanuts, cottonseed, and tree nuts.
2. Aspergillus flavus and Aspergillus parasiticus produce alfatoxins.
3. Exposure to aflatoxins is associated with an increased risk of liver cancer.

Which of the statements given above is/are correct?


A. 2 and 3 only
B. 1 and 2 only
C. 1, 2 and 3
D. None

Correct Answer : A

Answer Justification :

Aflatoxins are a family of toxins produced by certain fungi that are found on agricultural crops
such as maize (corn), peanuts, cottonseed, and tree nuts. Hence, statement 1 is incorrect.

The main fungi that produce aflatoxins are Aspergillus flavus and Aspergillus parasiticus,
which are abundant in warm and humid regions of the world. Hence, statement 2 is
correct.

Aflatoxin-producing fungi can contaminate crops in the field, at harvest, and during storage.

57
Total Marks : 200
Online Prelims TEST - 13 (SUBJECT WISE)
( InsightsIAS Mock Test Series for UPSC Preliminary Exam 2020 )

Exposure to aflatoxins is associated with an increased risk of liver cancer. Hence, statement 3 is
correct.

https://www.downtoearth.org.in/news/health/carcinogen-aflatoxin-detected-in-fssai-milk-survey-sam
ples-67327

https://www.cancer.gov/about-cancer/causes-prevention/risk/substances/aflatoxins

88 State of the World’s Children report published recently by which of the following organization?

A. UNESCO
B. OECD
C. World Bank
D. UNICEF

Correct Answer : D

Answer Justification :

The State of the World's Children is an annual report published by the United Nations Children's
Emergency Fund. It is the flagship publication of the organization.

https://www.unicef.org/reports/state-of-worlds-children-2019

89 Consider the following statements:


1. Operation Greens aims for integrated development of Tomato, Onion and Potato (TOP) value chain.
2. The Mega Food Park project is implemented by a Special Purpose Vehicle (SPV) which is a Body
Corporate registered under the Companies Act.

Which of the statements given above is/are correct?


A. 1 only
B. 2 only
C. Both 1 and 2
D. Neither 1 nor 2

Correct Answer : C

Answer Justification :

In the budget speech of Union Budget 2018-19, a new Scheme “Operation Greens” was
announced on the line of “Operation Flood”, with an outlay of Rs.500 crore to promote Farmer
Producers Organizations (FPOs #), agri-logistics, processing facilities and professional
management. Accordingly, the Ministry has formulated a scheme for integrated development
of Tomato, Onion and Potato (TOP) value chain. Hence, statement 1 is correct.

58
Total Marks : 200
Online Prelims TEST - 13 (SUBJECT WISE)
( InsightsIAS Mock Test Series for UPSC Preliminary Exam 2020 )

The Scheme of Mega Food Park aims at providing a mechanism to link agricultural production to
the market by bringing together farmers, processors and retailers so as to ensure maximizing value
addition, minimizing wastage, increasing farmers income and creating employment opportunities
particularly in rural sector. The Mega Food Park Scheme is based on “Cluster” approach and
envisages creation of state of art support infrastructure in a well-defined agri / horticultural zone
for setting up of modern food processing units in the industrial plots provided in the park with well-
established supply chain.

The Mega Food Park project is implemented by a Special Purpose Vehicle (SPV) which is a
Body Corporate registered under the Companies Act. State Government, State Government
entities and Cooperatives are not required to form a separate SPV for implementation of Mega Food
Park project. Hence, statement 2 is correct.

90 With reference to the Senior Citizens Savings Scheme (SCSS), consider the following statements:
1. Non-Resident Indians (NRIs) and Person of Indian Origin (PIOs) are entitled to open a Senior
Citizens Savings Scheme account.
2. Investments made in a Senior Citizen Savings Scheme account qualify for income tax deduction
benefit under Section 80C of the Income Tax Act, 1961.
3. It is available through Public / Private sector banks and India Post Offices.

Which of the statements given above is/are correct?


A. 1 only
B. 1 and 3 only
C. 2 and 3 only
D. 1, 2 and 3

Correct Answer : C

Answer Justification :

Senior Citizens Savings Scheme (SCSS) is a government-backed savings instrument offered to


Indian residents aged over 60 years. Non-Resident Indians (NRIs) and Person of Indian
Origin (PIOs) are not entitled to open a Senior Citizens Savings Scheme account.

The deposit matures after 5 years from the date of account opening but can be extended once by an
additional 3 years. Hence, statement 1 is incorrect.

SCSS is available through Public / Private sector banks and India Post Offices. Hence,
statement 3 is correct.

Being a government-backed savings instrument, the terms and conditions applicable to the SCSS
are the same, regardless of the bank/ post office you invest through.

Investments made in a Senior Citizen Savings Scheme account qualify for income
tax deduction benefit under Section 80C of the Income Tax Act, 1961. Hence, statement 2
is correct.

59
Total Marks : 200
Online Prelims TEST - 13 (SUBJECT WISE)
( InsightsIAS Mock Test Series for UPSC Preliminary Exam 2020 )

Interest on SCSS is fully taxable. In case the interest amount earned is more than Rs. 50,000 for
a fiscal, Tax Deducted at Source (TDS) is applicable to the interest earned.

https://www.paisabazaar.com/saving-schemes/senior-citizen-savings-scheme/#Eligibility_SCSS

91 The Participatory Guarantee Systems (PGS) sometimes seen in news is related to:

A. Organic quality assurance system


B. Ozone layer protection
C. Panchayat Raj
D. Wildlife sanctuaries

Correct Answer : A

Answer Justification :

The PGS is an internationally applicable organic quality assurance system [like ISO 9000]
implemented and controlled by the committed organic farmer-producers through active
participation, along with the consumers, in the process based on verifiable trust.

Hence, option (a) is correct.

http://www.pgsorganic.in/

https://indianexpress.com/article/explained/explained-what-is-pgs-the-heart-of-the-organic-food-prod
uction-industry-6021466/

92 With reference to the Index of Industrial Production (IIP) in India, consider the following statements:
1. The current base year for the IIP series in India is 2004-05.
2. It is compiled and published by the Central Statistics Office(CSO).
3. Mining sector has more weightage than Electricity sector in the index.

Which of the statements given above is/are correct?


A. 1 and 2 only
B. 2 and 3 only
C. 1 and 3 only
D. 1, 2 and 3

Correct Answer : B

Answer Justification :

Index of Industrial Production (IIP) measures the quantum of changes in the industrial production in
an economy and captures the general level of industrial activity in the country. It is a composite
indicator expressed in terms of an index number which measures the short term changes in the

60
Total Marks : 200
Online Prelims TEST - 13 (SUBJECT WISE)
( InsightsIAS Mock Test Series for UPSC Preliminary Exam 2020 )

volume of production of a basket of industrial products during a given period with respect to
the base period.

The current base year for the IIP series in India is 2011-12. Hence, statement 1 is
incorrect.

Index of Industrial Production is compiled and published every month by Central Statistics
Office (CSO) of the Ministry of Statistics and Programme Implementation with a time lag of six
weeks from the reference month. Hence, statement 2 is correct.

Industrial production for the purpose of IIP is divided into three sectors, i.e, Mining,
Manufacturing and Electricity. In 'Sectoral' classification, relative weights of
Manufacturing, Mining and Electricity are 75.5%, 14.2% and 10.3% respectively. Hence,
statement 3 is correct.

http://www.arthapedia.in/index.php?title=Index_of_Industrial_Production

https://economictimes.indiatimes.com/news/economy/indicators/august-iip-growth-slips-to-1-1-per-c
ent/articleshow/71541622.cms

93 ‘BepiColombo’, a spacecraft to mercury is joint mission of

A. ISRO and Sri Lankan Space agency


B. ISRO and European Space Agency (ESA)
C. The National Aeronautics and Space Administration (NASA) and European Space Agency
(ESA)
D. European Space Agency (ESA) and Japan Aerospace Exploration Agency (JAXA)

Correct Answer : D

Answer Justification :

BepiColombo is Europe's first mission to Mercury. It will set off in 2018 on a journey to the
smallest and least explored terrestrial planet in our Solar System. When it arrives at Mercury in late
2025, it will endure temperatures in excess of 350 °C and gather data during its 1 year nominal
mission, with a possible 1-year extension. The mission comprises two spacecraft: the Mercury
Planetary Orbiter (MPO) and the Mercury Magnetospheric Orbiter (MMO). BepiColombo is a
joint mission between ESA and the Japan Aerospace Exploration Agency (JAXA), executed
under ESA leadership.

Hence, option (d) is correct.

94 Consider the following statement:


1. Typbar TCV is the world’s first clinically proven conjugate Typhoid vaccine.
2. JENVAC is a single dose inactivated Japanese Encephalitis (JE) Vaccine.

61
Total Marks : 200
Online Prelims TEST - 13 (SUBJECT WISE)
( InsightsIAS Mock Test Series for UPSC Preliminary Exam 2020 )

Which of the statements given above is/are correct?


A. 1 only
B. 2 only
C. Both 1 and 2
D. Neither 1 nor 2

Correct Answer : C

Answer Justification :

Typbar TCV is the world’s first clinically proven conjugate Typhoid vaccine. Further, Typbar
TCV is the only approved vaccine for children and infants less than 2 years of age. Hence,
statement 1 is correct.

JENVAC is a single dose inactivated Japanese Encephalitis (JE) Vaccine. This Vero cell
derived vaccine is prepared from an Indian strain (Kolar- 821564XY) of the JE virus.
JENVAC has been developed in collaboration with the National Institute of Virology, India. It is a
safe and highly-effective vaccine that protects against all known strains of Japanese Encephalitis.
Hence, statement 2 is correct.

https://www.thehindu.com/sci-tech/science/can-typhoid-be-an-illness-of-the-past/article29667473.ec
e

95 Consider the following statements:


1. Global Hunger Index is calculated using undernourishment, child wasting, child stunting and Infant
mortality rate (IMR) indicators.
2. Compact2025 is an initiative for ending hunger and under nutrition by 2025.

Which of the statement given above is/are correct?


A. 1 only
B. 2 only
C. Both 1 and 2
D. Neither 1 nor 2

Correct Answer : B

Answer Justification :

Global Hunger Index (GHI) is a tool designed to comprehensively measure and track hunger at
global, regional, and national levels.

First, for each country, values are determined for four indicators:

1. UNDERNOURISHMENT: the share of the population that is undernourished (that is, whose
caloric intake is insufficient);

62
Total Marks : 200
Online Prelims TEST - 13 (SUBJECT WISE)
( InsightsIAS Mock Test Series for UPSC Preliminary Exam 2020 )

2. CHILD WASTING: the share of children under the age of five who are wasted (that is, who
have low weight for their height, reflecting acute undernutrition);

3. CHILD STUNTING: the share of children under the age of five who are stunted (that is, who
have low height for their age, reflecting chronic undernutrition); and

4. CHILD MORTALITY: the mortality rate of children under the age of five (in part, a reflection
of the fatal mix of inadequate nutrition and unhealthy environments).

It will not take IMR into consideration. Hence, statement 1 is incorrect.

Compact2025 is an initiative for ending hunger and undernutrition by 2025. It brings stakeholders
together to set priorities, innovate and learn, fine-tune actions, build on successes, and synthesize
sharable lessons in order to accelerate progress. Hence, statement 2 is correct.

https://indianexpress.com/article/explained/global-hunger-index-why-india-trails-6072465/

https://www.compact2025.org/about-compact2025/

96 Consider the following statements:


1. Nepal shares boundary with five Indian states.
2. The Sharda River demarcates Nepal's eastern border with India.
3. Surya Kiran is the military exercise between India and Nepal.

Which of the statements given above is/are correct?


A. 1 and 2 only
B. 2 and 3 only
C. 1 and 3 only
D. 1, 2 and 3

Correct Answer : C

Answer Justification :

The five Indian states that share a land border with Nepal are Uttarakhand, Uttar Pradesh,
Bihar, West Bengal, and Sikkim. Hence, statement 1 is correct.

The Sharda River demarcates Nepal's western border with India. It descends from 3,600 m at
Kalapani to 200 m as it enters the Terai plains in Uttar Pradesh, flowing southeast across the plains
to join the Ghaghra river, a tributary of the Ganges. Hence, statement 2 is incorrect.

Exercise SURYA KIRAN is a biannual event which is conducted alternatively in Nepal and India.
Notably in the series of military training exercises undertaken by India with various countries,
Exercise SURYA KIRAN with Nepal is the largest in terms of troop participation. Hence,
statement 3 is correct.

63
Total Marks : 200
Online Prelims TEST - 13 (SUBJECT WISE)
( InsightsIAS Mock Test Series for UPSC Preliminary Exam 2020 )

https://indianexpress.com/article/opinion/columns/china-nepal-xi-jinping-international-relations-secu
rity-diplomacy-6069020/

https://pib.gov.in/newsite/PrintRelease.aspx?relid=179508

97 Consider the following statements:


1. Lake Van is located in Turkey.
2. Sea of Marmara connects the Black Sea to the Aegean Sea.
3. The Bosporus forms part of the continental boundary between Europe and Asia.

Which of the statements given above is/are correct?


A. 1 only
B. 2 and 3 only
C. 3 only
D. 1, 2 and 3

Correct Answer : D

Answer Justification :

Lake Van, Turkish Van Gölü, lake, largest body of water in Turkey and the second largest in
the Middle East. The lake is located in the region of eastern Anatolia near the border
of Iran. Hence, statement 1 is correct.

The Sea of Marmara, also known as the Sea of Marmora or the Marmara Sea, is the
inland sea that connects the Black Sea to the Aegean Sea. It separates Turkey's Asian and
European parts. Hence, statement 2 is correct.

The Bosporus or Bosphorus is a narrow, natural strait and an internationally significant waterway
located in northwestern Turkey. It forms part of the continental boundary between Europe and
Asia, and divides Turkey by separating Anatolia from Thrace. Hence, statement 3 is correct.

https://www.thehindu.com/opinion/op-ed/a-turkish-misadventure/article29683319.ece

98 Consider the following statements


1. India Innovation Index (III) is a new index launched by NITI Aayog only
2. The index’s objective is to rank Indian states on innovations through the portal that will capture
data on innovation from all states on innovation and regularly update it in real time
3. It will also ensure progress towards innovation at the local level in India.

Which of the statements given above is/are correct?


A. 2 only
B. 1 and 2 only
C. 2 and 3 only
D. 1, 2 and 3

64
Total Marks : 200
Online Prelims TEST - 13 (SUBJECT WISE)
( InsightsIAS Mock Test Series for UPSC Preliminary Exam 2020 )

Correct Answer : C

Answer Justification :

India Innovation Index (III):

NITI Aayog with Institute for Competitiveness as the knowledge partner released the India
Innovation Index (III) 2019. Hence, statement 1 is incorrect.

The index’s objective is to rank Indian states on innovations through the portal that will
capture data on innovation from all states on innovation and regularly update it in real time.
Hence, statement 2 is correct.

The index has been jointly developed by NITI Aayog, DIPP and CII in consultation with World
Economic Forum (WEF), World Intellectual Property Organization (WIPO), Cornell University,
UNIDO, ILO, OECD, UNESCO, ITU etc.

It will also ensure progress towards innovation at the local level in India. Hence, statement
3 is correct.

99 Consider the following statements


1. NITI Aayog has released the 20th Livestock Census report recently.
2. West Bengal observed the highest increase of 23%, followed by Telangana (22%).
3. For the first time, livestock data has been collected online through tablet computers.

Which of the statements given above is/are correct?


A. 1 and 2 only
B. 2 and 3 only
C. 1 and 3 only
D. 1, 2 and 3

Correct Answer : B

Answer Justification :

Ministry of Fisheries, Animal Husbandry and Dairying has released the 20th Livestock Census
report recently. Hence, statement 1 is incorrect.

The total Livestock population is 535.78 million in the country showing an increase of 4.6%
over Livestock Census-2012.

65
Total Marks : 200
Online Prelims TEST - 13 (SUBJECT WISE)
( InsightsIAS Mock Test Series for UPSC Preliminary Exam 2020 )

West Bengal observed the highest increase of 23%, followed by Telangana (22%). Hence,
statement 2 is correct.

For the first time, livestock data has been collected online through tablet computers. Hence,
statement 3 is correct.

https://indianexpress.com/article/explained/reading-the-livestock-census-2019-india-6075270/

100 The Ashok Dalwai committee is related to

A. Non-performing assets
B. Public undertakings
C. Women empowerment
D. Doubling the farmers’ income

Correct Answer : D

Answer Justification :

The Ashok Dalwai committee is related “doubling farmers” income is achieved by 2022. Hence,
option (d) is correct.

66

You might also like